ORTHOPEDIC MCQS ONLINE QUESTION BANK H2A
2072. (2226) Q5-2654:
The protein neurofibromin normally acts in which of the following ways:
1) Inhibits fibroblast growth factor
3) Downregulates Ras protein
2) Promotes proteoglycan assembly
5) Promotes tumor formation
4) Causes nerve cells to divide
If defective, neurofibromin is the protein that causes neurofibromatosis. Neurofibroma is coded on chromosome 17, and it acts as a tumor suppressor by downregulating Ras protein, which enhances cell growth and proliferation.Correct Answer: Downregulates Ras protein
2073. (2227) Q5-2655:
According to National Institutes of Health (NIH) criteria, what is the minimum number of 15-mm café-au-lait macules required as a diagnostic criterion for neurofibromatosis in postpubertal patients:
1) One
3) Three
2) Two
5) Six
4) Four
The NIH criteria require at least six 15-mm café-au-lait macules in postpubertal patients. Café-au-lait macules must be larger than 5 mm in prepubertal patients.Correct Answer: Six
2074. (2228) Q5-2656:
A 7-year-old girl presents with an acute fracture of her proximal radial metaphysis. Although a line down the shaft of the radius intersects the center of the capitellum, the articular surface of the radial head is angled 20° from the anatomic position as compared with the other elbow. You recommend:
1) Observation with follow-up in 1 week
3) Closed manipulation with a supination maneuver
2) Closed manipulation with a pronation maneuver
5) Open reduction and internal fixation
4) Closed manipulation with a percutaneous K-wire
Proximal radial fractures have excellent remodeling potential, especially if the angulation is less than 30°. Manipulation is not necessary, and the risk of stiffness from any invasive procedure is not worthwhile.Correct Answer: Observation with follow-up in 1 week
2075. (2229) Q5-2657:
Which of the following findings is typical in patients with Marfan syndrome as opposed to patients with Ehlers-Danlos syndrome:
1) Scoliosis
3) Pes planus
2) Dural ectasia
5) Joint pains
4) Lens dislocation
All of the findings presented, with the exception of lens dislocation, are seen in both syndromes at a frequency that exceeds the general population. Lens dislocation is common in patients with Marfan syndrome but not those with Ehlers-Danlos syndrome.Correct Answer: Lens dislocation
2076. (2230) Q5-2658:
Ehlers-Danlos syndrome is caused by a defect in which of the following:
1) Elastin
3) Dermatan sulfate
2) Collagen
5) Fibroblast growth factor
4) Proteoglycan
There are 11 subtypes of Ehlers-Danlos syndrome. Each of the subtypes is caused by defects in collagen types 1, 3, or 5, or their processing. Defects in a component of elastic microfibrils (fibrillin) are responsible for Marfan syndrome. Defects in dermatan sulfate processing cause Hurler syndrome. Defects in fibroblast growth factor cause achondroplasia. Defects in proteoglycan processing cause diastrophic dysplasia.
Correct Answer: Collagen
2077. (2231) Q5-2659:
A 4-year-old boy with macrodactyly of the foot has involvement of the second and third rays. He undergoes debulking of the soft tissues of the phalanges and amputation of the distal phalanges. Two years later, he returns with a dramatic increase in the width and length of the involved regions. You recommend:
1) Epiphyseodesis of the phalanges and metatarsals
3) Ray resection of one of the involved rays with further debulking
2) Amputation of the proximal phalanges
5) Syme amputation
4) Chopart amputation
The increase in width of the involved regions can be handled only by ray resection. Additional proximal levels of amputation are not required.Correct Answer: Ray resection of one of the involved rays with further debulking
2078. (2232) Q5-2660:
A 14-year-old girl is kicking a soccer ball when she feels a âpopâ in her hip. The most likely diagnosis is:
1) Slipped sacroiliac joint
3) Pathologic hip fracture
2) Slipped capital femoral epiphysis
5) Ligamentum teres avulsion
4) Avulsion of the anterior inferior iliac spine
This is a classic description of avulsion of the anterior inferior iliac spine. When this patient hyperextended her hip and flexed her knee simultaneously to kick a ball, the rectus femoris was stretched at both joints. In a skeletally immature patient, this apophysis is not fully ossified and is vulnerable to avulsion. Treatment is symptomatic, with return to sports in about 6 weeks.Correct Answer: Avulsion of the anterior inferior iliac spine
2079. (2233) Q5-2661:
A 3-year-old boy has a progressive anterior bow of the right tibia. He experiences intermittent aching. His physical examination is otherwise unremarkable. Radiographs reveal 25° of anterior bow of the tibia just distal to the midshaft and 20° of lateral bow. There is some narrowing of the medullary canal around the bow and thinning of the anterior cortex. You recommend which course of action:
1) Floor reaction ankle-foot orthosis
3) Osteotomy and plate fixation
2) Osteotomy and intramedullary rod
5) Free vascularized fibular graft
4) Tibiofibular synostosis
This patient has a congenitally dysplastic tibia. The tibia is at risk for fracture, but orthotic protection is sometimes successful in preventing fracture. Operative intervention should be reserved for fracture because there is a significant risk of nonunion or delayed union.Correct Answer: Floor reaction ankle-foot orthosis
2080. (2234) Q5-2662:
An 8-year-old patient with cerebral palsy has an equinovarus foot on the right side. The varus is worse during push-off. He also holds his right upper extremity stiffly when he walks. He is developing a pressure callus on the lateral side of his foot in the region of the calcaneocuboid joint. Passively, the foot can be corrected to a neutral position of varus-valgus but lacks 12° from neutral dorsiflexion. Your recommendation is:
1) Ankle-foot orthosis (AFO)
3) Lengthening of the triceps surae and triple arthrodesis
2) Lengthening of the triceps surae
5) Lengthening of the triceps surae and split posterior tibial tendon transfer
4) Tendoachilles lengthening and peroneal shortening
An equinovarus foot is commonly found in patients with hemiplegic cerebral palsy. The varus aspect is difficult to brace. In this patient with fixed equinus, an AFO would not be a successful treatment option. A lengthening of the triceps surae would be indicated, and it could be done at the level of the Achilles tendon or by a gastrocnemius-soleus recession, depending on the clinical examination. In addition, another procedure is necessary to deal with the varus. The best way to accomplish this would be by split posterior tibial tendon transfer. This patient is too young for a triple arthrodesis, as this procedure is appropriate in patients at or near skeletal maturity with rigid deformity.
Correct Answer: Lengthening of the triceps surae and split posterior tibial tendon transfer
2081. (2235) Q5-2663:
Hip subluxation is most commonly seen in which type of cerebral palsy:
1) Spastic hemiplegia
3) Spastic triplegia
2) Spastic diplegia
5) Athetoid
4) Total involvement
Neuromuscular subluxation of the hips is caused by muscle imbalance over time. It is most common in patients with greatest imbalance â total body involvement. Hip subluxation is rare in patients with spastic hemiplegia.
Correct Answer: Total involvement
2082. (2236) Q5-2664:
Which group of cerebral palsy patients is most likely to sustain a pathologic fracture:
1) Spastic hemiplegic
3) Spastic triplegic
2) Spastic diplegic
5) Athetoid
4) Total involvement
Pathologic fracture is most common in cerebral palsy patients with total involvement, presumably because they have little stress on the bone from either muscle tone or standing.
Correct Answer: Total involvement
2083. (2237) Q5-2665:
Which is the most common location of pathologic fracture in patients with cerebral palsy:
1) Radius
3) Humerus
2) Femur
5) Calcaneus
4) Tibia
Pathologic fracture is a common problem in patients with cerebral palsy. Pathologic fracture most commonly involves the femur, especially the shaft and distal metaphysis. This should be the first diagnostic consideration in a nonambulatory patient with a warm or swollen knee.Correct Answer: Femur
2084. (2238) Q5-2666:
A 5-year-old girl is brought to the emergency department because of fever and inability to walk. Her temperature is 100.5° F. She has pain with rotation of the hip. However, if the movement is done slowly, the hip can be rotated internally and externally 45°. Her white blood cell count is 13,000 (upper normal is 12,500 for her age). Her erythrocyte sedimentation rate is 30.
Radiographs of the pelvis and hip are normal. You recommend:
1) Bed rest with follow-up contact the next day
3) Hip aspiration
2) Ultrasound of the hip
5) Irrigation and drainage of the hip
4) Magnetic resonance imaging
The scenario described above is most consistent with transient synovitis of the hip. Because range of motion is tolerated when performed slowly; it is compatible with a diagnosis of transient synovitis of the hip more than infection. Rest, with or without anti-inflammatory medicine, should produce a dramatic improvement by the next day.
Correct Answer: Bed rest with follow-up contact the next day
2085. (2239) Q5-2667:
Which of the following factors is least likely to predispose a patient to patellar instability:
1) Genu varum
3) Patella alta
2) Excessive femoral anteversion
5) Hypoplastic lateral femoral trochlear ridge
4) External tibial torsion
Genu varum does not predispose a patient to patellar instability. However, genu valgum, as well as all of the other factors listed above, may predispose a patient to this condition.
Correct Answer: Genu varum
2086. (2240) Q5-2668:
A 15-year-old girl twists her knee while skiing. She is diagnosed with a patellar dislocation. She has had no prior episodes. Radiographs show the dislocation but no other findings. After reducing the dislocation, you recommend:
1) Knee immobilization for 2 to 3 weeks
3) Repair of the medial patellofemoral ligament
2) Long-leg cast for 6 weeks
5) Proximal realignment
4) Tibial tubercle transfer
Acute traumatic patellofemoral dislocations are best treated by brief immobilization followed by early rehabilitation. Surgery is reserved for patients with multiple recurrences that cause significant disability.
Correct Answer: Knee immobilization for 2 to 3 weeks
2087. (2241) Q5-2669:
Which of the following is related to the etiology of Ewingâs sarcoma:
1) Chromosomal translocation (11/22)
3) Bacterial infection
2) Viral infection
5) Single base mutation
4) Trauma
Ewingâs sarcoma belongs to the family of round cell sarcomas. A translocation t(11:22)(q24q12) has been discovered in Ewingâs sarcoma. This translocation results in a novel protein, EWS-FL11, that causes rapid growth of cells arising from the neural crest.
Correct Answer: Chromosomal translocation (11/22)
2088. (2242) Q5-2670:
A 5-year-old girl is brought to your office because of a mass in the back of her right knee. The family has noticed the mass for the past 8 months, and they also tell you that it varies in size with activity. The mass is painless and is located on the medial side of the popliteal fossa. It measures 2 cm to 4 cm and is discrete, firm, and nontender. Examinations of her knee and gait are unremarkable otherwise. You recommend:
1) Observation with return if any changes
3) Aspiration
2) Magnetic resonance imaging
5) Knee arthroscopy to look for intra-articular causes
4) Excisional biopsy
The patient history and physical examination are consistent with a benign popliteal cyst. This type of cyst is usually unassociated with any intra-articular pathology in children. The cyst resolves on its own with time. Further study or biopsy would be indicated only if the mass were progressively enlarging, tender, or located in an atypical area.
Correct Answer: Observation with return if any changes
2089. (2243) Q5-2671:
A 20-month-old toddler is brought in because of bowed legs. You note moderate-to-severe genu varum and thigh-foot angles of 35° inward. The child is otherwise healthy, and height and weight are near the 50th percentiles. Radiographs demonstrate tibial metaphyseal-diaphyseal angles of 8° on the right and 9° on the left. Femoral metaphyseal-diaphyseal angles are the same.
You recommend:
1) Observation, with return only if not improving
3) Serial casting until straight
2) Measurement of electrolytes, alkaline phosphatase, and vitamin D levels
5) Osteotomies of the proximal tibias
4) Bracing in knee-ankle-foot orthoses
This child has physiologic genu varum. Only if the metaphyseal-diaphyseal angles were greater than 11° would additional followup be indicated, with possible bracing. Blood tests would be indicated if the bone quality showed evidence of Rickets or if the child was below the 10th percentile in height.
Correct Answer: Observation, with return only if not improving
2090. (2244) Q5-2672:
Which of the following is the most common complication after a posttraumatic distal radial growth arrest:
1) Positive distal ulnar variance
3) Increased distal radial articular angle
2) Negative distal ulnar variance
5) Distal radioulnar joint instability
4) Decreased distal radial articular angle
The most common complication after a posttraumatic distal radial growth arrest is positive ulnar variance (overgrowth). Other complications may include decreased distal radial articular angle, triradiate fibrocartilage tear, or distal radioulnar joint instability, but they are less common. Increased radial articular angle and negative ulnar variance rarely develop after these injuries.
Correct Answer: Positive distal ulnar variance
2091. (2245) Q5-2673:
Patients with which one of the following curve types associated with idiopathic scoliosis are at increased risk of shortness of breath:
1) Thoracic curves larger than 80°
3) Thoracolumbar curves larger than 100°
2) Thoracolumbar curves larger than 80°
5) Lumbar curves larger than 100°
4) Lumbar curves larger than 80°
Idiopathic scoliosis is associated with measurable decreases in pulmonary function in thoracic curves larger than 60° to 70°, but a clinically significant increase in risk of shortness of breath has been shown only in thoracic curves larger then 80°.
Correct Answer: Thoracic curves larger than 80°
2092. (2246) Q5-2674:
Which of the following patients with infantile idiopathic scoliosis can be observed without a magnetic resonance image (MRI):
1) A patient with a curve less than 20°
3) A patient with a right thoracic curve
2) A patient with a normal neurologic examination
5) A patient without any pain
4) A patient younger than 2 years old
Patients with idiopathic infantile scoliosis have a 22% incidence of abnormalities that can be viewed on MRI. Syrinx and Chiari malformation are the most common abnormalities, with a similar rate as that found in patients with juvenile idiopathic scoliosis. The left thoracic curve pattern is most commonly seen in infantile patients, but a right thoracic curve is not protective. Age, freedom from pain, or a normal neurologic examination are also not protective. Magnetic resonance imaging is recommended for all infantile curves larger than 20°.
Correct Answer: A patient with a curve less than 20°
2093. (2247) Q5-2675:
Which of the following features differentiates a grade 2 open fracture from a grade 1 open fracture:
1) Laceration larger than 1 mm
3) Laceration larger than 1 in
2) Laceration larger than 1 cm
5) Associated vascular injury
4) Associated neurologic injury
A grade 2 open fracture is distinguished from a grade 1 open fracture by a laceration larger than 1 cm. Grade 3 open fractures are characterized by massive soft tissue damage, circulatory compromise, severe contamination, or marked instability. Grade 3A open fracture characteristics include extensive soft tissue lacerations but adequate skin to cover the bone. Grade 3B fractures show extensive soft tissue loss. Grade 3C open fractures demonstrate arterial injury and require repair.
Correct Answer: Laceration larger than 1 cm
2094. (2248) Q5-2676:
Which of the following features differentiates a grade 3B open fracture from a grade 3C open fracture:
1) Presence of neurologic injury
3) Presence of vascular injury not requiring repair
2) Presence of vascular injury requiring repair
5) Massive contamination
4) Segmental bone loss
Grade 3 injuries all have massive soft tissue damage. Grade 3C fractures have vascular injury requiring repair, whereas grade 3B fractures do not have vascular injury or do not require repair. An example of the latter is a severe open fracture of the distal tibia with laceration of the dorsalis pedis artery and a foot that is well perfused through its collaterals.
Correct Answer: Presence of vascular injury not requiring repair
2095. (2249) Q5-2677:
Which rating best describes a childâs hip affected with Perthesâ disease that has healed with aspherical incongruity:
1) Catterall 3
3) Herring C
2) Catterall 4
5) Stulberg 5
4) Stulberg 4
The Catterall and Herring classifications are used during the evolution of Perthesâ disease to guide treatment; they are used during the fragmentation stage. It is impossible to know what the Catterall or Herring class was after healing has occurred. The Stulberg classification gives prognosis after healing. Stulberg 4 is aspherical but congruous. Stulberg 5 is aspherical and incongruous. Patients classified with Stulberg 5 hips typically have degenerative joint disease in early adulthood.
Correct Answer: Stulberg 5
2096. (2250) Q5-2678:
Which of the following agents is used to reverse the effects of midazolam:
1) Naloxone
3) Flumazenil
2) Naltrexone
5) Atropine
4) Fentanyl
Flumazenil (reversed) is used to reverse the effects of benzodiazepines. Flumazenil has a shorter duration of action than benzodiazepines, so it may need to be readministered. Flumazenil can also precipitate seizures.Correct Answer: Flumazenil
2097. (2251) Q5-2679:
A 6-year-old boy is receiving pharmacologic agents to assist in the reduction of a forearm fracture. Which of the following conditions is not necessary for him to be in âconscious sedationâ:
1) His consciousness is fully normal.
3) His airway reflexes are maintained.
2) He can open his eyes to command.
5) His vital signs are stable.
4) He maintains a patent airway.
Under conscious sedation, a child must be able to open eyes to command, maintain a patent airway and reflexes, and have stable vital signs. However, consciousness is medically depressed by the drugs.
Correct Answer: His consciousness is fully normal.
2098. (2252) Q5-2680:
A mutation in type II collagen is responsible for all of the following conditions except:
1) Stickler syndrome
3) Kniest syndrome
2) Spondyloepiphyseal dysplasia congenita
5) Achondroplasia
-
Strudwick dysplasia
Type II collagen is largely found in hyaline cartilage. All of the dysplasias, with the exception of achondroplasia, have significant abnormalities of articular cartilage. Achondroplasia results from a defect in fibroblast growth factor receptor protein.
Correct Answer: Achondroplasia
2099. (2253) Q5-2681:
An adult man with hemophilia A has just announced the birth of his first son. His wife does not have the disease. What is the chance that the newborn has the disorder:
1) 100%
3) 25%
2) 50%
-
Less than 1%
4) 10%
Hemophilia A is an X-linked disorder. Therefore, there is no father-son transmission. The chance that the newborn is affected equals that of the general population, which is less than 1%.
Correct Answer: Less than 1%
2100. (2254) Q5-2682:
Which of the following conditions is least commonly seen in patients with congenital dislocation of the patella:
-
Fibrosis of the vastus lateralis
3) Hypoplasia of the vastus medialis
2) Contracture of the iliotibial band
5) Flexion contracture
4) Enlarged patella
Congenital dislocation of the patella is present from birth and diagnosed in childhood. It is not reducible with the knee in extension. Essential elements include contracture of the iliotibial band, vastus lateralis, hypoplasia of the vastus medialis, lateral insertion of the patellar tendon, decreased size of the patella, flexion contracture and valgus alignment of the knee, and hypoplasia of the patellar sulcus. Operative treatment is usually successful, and the success rate is increased with early surgery.
Correct Answer: Enlarged patella
2101. (2255) Q5-2683:
The surgical reconstruction of a congenitally dislocated patella includes all of the following elements except:
1) Advancement of the vastus medialis insertion
3) Lengthening or release of the biceps femoris muscle
2) Lengthening or release of the iliotibial band
5) Lateral transfer of the patellar tendon insertion
4) Lateral capsular release
The surgical reconstruction of a congenitally dislocated patella includes lengthening or release of the lateral capsule, lengthening or release of the iliotibial band, lengthening or release of the biceps femoris muscle, distal advancement or imbrication of the patellar tendon, and sometimes a rectus femoris lengthening. Semitendinosus transfer to the patella may also be added. The lateral insertion of the patellar tendon may be transferred medially.
Correct Answer: Lateral transfer of the patellar tendon insertion
2102. (2256) Q5-2684:
Which of the following radiographic parameters is most appropriately used to assess the status of the hip in a growing child with cerebral palsy:
1) The Stulberg rating
3) The alpha and beta angle of Graf
2) The migration index of Reimer
5) The Hilgenreiner-epiphyseal angle
4) Kleinâs line
The migration index (of Reimer) is the percentage of the femoral head outside of the acetabulum. This radiographic parameter tracks the progress of the hip in patients with cerebral palsy because the femoral head may gradually migrate out.
The other parameters refer to radiographic assessments of other hip disorders: The Stulberg rating is used in patients with healed Perthesâ disorder.
The alpha and beta angles of Graf provide ultrasonic assessment of patients with developmental hip dysplasia. Kleinâs line is used in early detection of slipped capital femoral epiphysis.
The Hilgenreiner-epiphyseal angle is used in developmental coxa vara.
Correct Answer: The migration index of Reimer
2103. (2257) Q5-2685:
What is the primary force applied to the spine during a Chance (seatbelt) fracture:
1) Flexion
3) Rotation
2) Compression
5) Flexion and rotation
4) Distraction
The primary force applied to the spine during a Chance fracture is distraction. The body is flexed forward around an axis in front of the body (often a seatbelt), but the spine is subject to distraction because it is posterior to this axis. This motion results in a characteristic distraction of bony or ligamentous elements with minimal crush.
Correct Answer: Distraction
2104. (2258) Q5-2686:
Which of the following physical findings is uncommon in a patient with a unilateral untreated clubfoot:
1) Thin calf
3) Plantar-medial prominence of the talus in the foot
2) Loss of prominence of the heel
5) Decreased length of the foot
4) Medial midfoot crease
In a patient with unilateral untreated clubfoot, certain findings are apparent when compared with the unaffected foot. Common findings include a thinner calf, shorter-appearing foot, loss of heel prominence, and a medial midfoot crease. The talus is prominent anterolaterally in the foot but not on the plantar-medial side.
Correct Answer: Plantar-medial prominence of the talus in the foot
2105. (2259) Q5-2687:
Which of the following radiographic findings is present in a varus foot:
1) Increased parallelism of the talus and calcaneus on lateral film
3) Increased divergence of the talus and calcaneus on the anteroposterior film
2) Superimposition of the anterior portions of the talus and calcaneus on the lateral film
5) Increased superimposition of the metatarsal heads on lateral film
4) Increased abduction of the forefoot on the hindfoot
Plain radiographs of a foot that is in varus show increased parallelism of the talus and the calcaneus on both the anteroposterior and lateral films. There is decreased superimposition of the talus, the calcaneus, and the metatarsal heads. By contrast, in a valgus foot, there is increased divergence on both the anteroposterior and lateral films.
Correct Answer: Increased parallelism of the talus and calcaneus on lateral film
2106. (2260) Q5-2688:
Although all of the following conditions may present in patients with skeletal dysplasia, which is the only condition that does not have a high frequency of spontaneous improvement:
1) Infantile foramen magnum stenosis in patients with achondroplasia
3) Cervical kyphosis in patients with diastrophic dysplasia
2) Infantile thoracolumbar kyphosis in patients with achondroplasia
5) Hypotonia in patients with achondroplasia
4) Cervical kyphosis in patients with Larsen syndrome
Cervical kyphosis does not improve in patients with Larsen syndrome and may cause myelopathy. Early arthrodesis (posterior only, if not too severe) is indicated. All of the other conditions have a high incidence of spontaneous improvement, so they are most appropriately treated with observation. In a minority of cases, the deformity does not improve and surgery is indicated.
Correct Answer: Cervical kyphosis in patients with Larsen syndrome
2107. (2261) Q5-2689:
All of the following conditions may affect lower extremities in children. Which one of the conditions does not have a high incidence of spontaneous improvement:
1) Posteromedial bow of the tibia
3) Metatarsus adductus
2) Calcaneovalgus foot
5) Anterolateral bowing of the tibia
4) Internal tibial torsion
Anterolateral bowing of the tibia is usually associated with tibial dysplasia and may progress to pseudarthrosis. Anterolateral bowing of the tibia does not resolve in most patients. Posteromedial bowing of the tibia usually resolves, although there may be a residual shortening of the tibia. All of the other conditions often completely resolve.
Correct Answer: Anterolateral bowing of the tibia
2108. (2262) Q5-2690:
The anterior interosseous nerve enables:
1) Sensation and flexion of the thumb and index fingers
3) Flexion of the lateral (ulnar) two fingers
2) Flexion of the thumb and index fingers
5) Abduction of the thumb
4) Flexion and sensation of the ulnar two fingers
The anterior interosseous nerve does not carry any sensory fibers. The anterior interosseous nerve enables flexion of the thumb (flexor digitorum pollicis) and index fingers (flexor digitorum profundus). This is the most commonly injured nerve in a supracondylar fracture, and it nearly always spontaneously recovers.
Correct Answer: Flexion of the thumb and index fingers
2109. (2263) Q5-2691:
Closed reduction without internal fixation is most likely to produce a satisfactory result in which of the following types of supracondylar fracture:
1) An intact posterior hinge and 20° of hyperextension
3) Posterior and medial translation with no intact hinge
2) An intact lateral hinge and 15° of varus
5) Anterior translation with 10° of varus
4) Posterior and lateral translation with no intact hinge
There are many options when treating a supracondylar fracture, but a surgeon should always choose the method with the highest percentage of good results for a given fracture. Answer A describes a type II supracondylar fracture with hyperextension and no varus-valgus displacement. The posterior hinge is intact, and it is relatively simple to reduce the fracture with flexion, immobilizing it at 120°. For the other fractures listed as possible answer choices, the reduction as well as the assessment of reduction is more complex. Therefore, most experts would prefer percutaneous fixation of these fractures once reduced.
Correct Answer: An intact posterior hinge and 20° of hyperextension
2110. (2264) Q5-2692:
The highest rate of success in restoring growth after physeal bar resection is found in which location:
1) Proximal femur
3) Proximal tibia
2) Distal femur
5) Proximal humerus
4) Distal tibia
The highest rate of success after physeal bar resection is found in the distal tibia. The mean growth after resection in the series by Peterson was 93% of that expected, which is greater than the success rates of the other locations. The reasons for this remain speculative but may be due to the more peripheral location of the bar at the ankle, less complex physeal shape, and lower energy of injury. Physeal bars in the proximal femur and humerus are rarely surgically treated.
Correct Answer: Distal tibia
2111. (2265) Q5-2693:
Which of the following fracture types are most prone to nonunion or refracture in patients with osteogenesis imperfecta:
1) Supracondylar humeral fractures
3) Supracondylar femur fractures
2) Olecranon apophysis fractures
5) Calcaneal fractures
4) Tibial fractures
Unless they are internally fixed, olecranon fractures are notoriously prone to nonunion or refracture in patients with osteogenesis imperfecta. The remaining fractures can be treated nonoperatively with cast immobilization.
Correct Answer: Olecranon apophysis fractures
2112. (2266) Q5-2694:
An 11-year-old boy has multiple, hard, tender nodules in his posterior shoulder girdle and paraspinous muscles. He also has a shortened first metatarsal with a delta-shaped first proximal phalanx. The diagnosis is:
1) Dysplasia epiphysialis hemimelica
3) Fibrodysplasia ossificans progressiva
2) Multiple exostoses
5) Metastatic osteosarcoma
4) Myositis ossificans
Fibrodysplasia ossificans progressiva is a disorder of the BMP4 signaling pathway. This condition is characterized by multiple osseous masses occurring in muscle, especially in the dorsal and axial regions. There is also a characteristic dysmorphology of the great toe, consisting of a shortened first metatarsal, delta-shaped phalanx, and some valgus.
Correct Answer: Fibrodysplasia ossificans progressiva
2113. (2267) Q5-2695:
Klippel-Trenaunay syndrome is characterized by cutaneous capillary venous malformation, varicose veins, and which of the following:
1) Multiple enchondromas
3) Limb hypertrophy
2) Multiple osteochondromas
5) Congenital pseudarthrosis of the tibia
4) Seizure disorder
Klippel-Trenaunay syndrome has three essential features: cutaneous capillary venous malformation, varicose veins, and hypertrophy of the involved limb in length and/or width. The syndrome usually presents before adolescence. The tissue abnormalities typically do not cross the midline, and the entire limb is not uniformly affected.
Correct Answer: Limb hypertrophy
2114. (2268) Q5-2696:
Which of the following characterizes Russell-Silver syndrome:
1) Hemihypertrophy
3) Acromegaly
2) Hemiatrophy
5) Precocious puberty
4) Macrodactyly
Russell-Silver syndrome is characterized by a small triangular face, hemiatrophy of up to 5 cm, delay in skeletal age, and sometimes a developmental delay. Scoliosis is commonly found in patients with Russell-Silver syndrome.
Correct Answer: Hemiatrophy
2115. (2269) Q5-2697:
Macrodactyly is associated with all of the following syndromes except:
1) Neurofibromatosis
3) Klippel-Trenaunay syndrome
2) Proteus syndrome
5) Multiple enchondromatosis
4) Marfan syndrome
Macrodactyly is an increase in length and width of a digit or two adjacent digits. It may be associated with all of the above syndromes except for Marfan syndrome. Macrodactyly is also a common isolated condition.
Correct Answer: Marfan syndrome
2116. (2270) Q5-2698:
Which of the following conditions is inherited in an X-linked dominant fashion:
1) Hemophilia A
3) Marfan syndrome
2) Familial hypophosphatemic rickets
5) Sickle cell anemia
4) Prader-Willi syndrome
Familial hypophosphatemic rickets is inherited in an X-linked dominant fashion, meaning that there is an excess of affected females. Hemophilia A is sex-linked recessive like most X-linked conditions (seen primarily in males). Marfan syndrome is autosomal dominant, sickle cell anemia is autosomal recessive, and Prader-Willi syndrome is a partial deletion of chromosome 15.
Correct Answer: Familial hypophosphatemic rickets
2117. (2271) Q5-2699:
Which of the following statements is true regarding congenital pseudarthrosis of the tibia:
1) Congenital pseudarthrosis of the tibia is nearly always associated with neurofibromatosis.
3) Congenital pseudarthrosis of the tibia usually presents with a valgus deformity.
2) Congenital pseudarthrosis of the tibia usually occurs in the proximal one-third of the tibia.
5) The surrounding bone is usually dysplastic.
4) Congenital pseudarthrosis of the tibia usually presents with an apex-posterior bow.
Congenital pseudarthrosis of the tibia is associated with neurofibromatosis in approximately one-half of patients. It usually occurs in the distal portion of the tibia, presents with a varus position, and an apex-anterior bow. The surrounding bone is usually dysplastic, displaying segments of sclerosis, tapering, and/or cystic changes. The fibula is often dysplastic as well.
Correct Answer: The surrounding bone is usually dysplastic.
2118. (2272) Q5-2700:
Which of the following elements is most critical to the success of the Williams rod procedure (intramedullary rod fixation) for congenital pseudarthrosis of the tibia:
1) Fibular fixation
3) Prolonged orthotic protection
2) Leaving the rod across the ankle at the time of surgery
5) Ultrasound stimulation
-
Electrical stimulation
Fixation of the fibula allows shortening and compression of the tibia, as well as providing mechanical stability. Neither leaving the rod across the ankle nor prolonged orthotic protection beyond the initial postoperative period appears to be essential. Electrical stimulation does not have a quantifiable effect on the success of the procedure. Ultrasound has not been studied in this condition.
Correct Answer: Fibular fixation
2119. (2273) Q5-2701:
Which of the following is considered the cutoff for normal loss of C 1-C 2 articular contact in dynamic computed tomography (CT) in normal patients:
1) 10%
3) 40%
2) 20%
-
80%
4) 50%
Up to 80% loss of contact between the C 1 and C 2 facet joints may be seen in normal patients. Therefore, dynamic CT is an oversensitive test and the dynamic feature does not add to CT in diagnosis.
Correct Answer: 80%
2120. (2274) Q5-2702:
In the absence of surgery, which of the following gait parameters in children with cerebral palsy tends to remain static with increasing age:
-
Knee range of motion
3) Normalized walking velocity
2) Ankle range of motion
5) Knee stiffness
4) Femoral anteversion
Most parameters of gait deteriorate over time in the absence of intervention in children with cerebral palsy. Knee and ankle range of motion decrease, as does walking velocity when normalized for height. Femoral anteversion remains unchanged. Knee stiffness increases.
Correct Answer: Femoral anteversion
2121. (2275) Q5-2703:
In patients with cerebral palsy, which of the following factors usually leads to an improvement in walking ability:
1) Increased body mass index
3) Increased age
2) Increased external tibial torsion
5) Alignment of the foot rocker with the line of progression
4) Valgus of the ankle
Gait in patients with cerebral palsy declines with increases in age, body mass index, external tibial torsion, and significant ankle valgus (due to worsened mechanics of pushoff). Gait is improved by aligning the foot rocker with the line of progression.
Correct Answer: Alignment of the foot rocker with the line of progression
2122. (2276) Q5-2704:
Nonselective nonsteroidal anti-inflammatory drugs (NSAIDs) have the following effect on spinal fusion:
1) Increased speed of fusion
3) Decreased bleeding
2) Increased volume of fusion bone
5) Unplanned extension of fusion
4) Decreased fusion rate
Nonselective NSAIDs have been shown to decrease fusion rates in both animal models and in humans. The mechanism is most likely through an inhibition of cyclooxygenase (COX)-1 because COX-2 inhibitors do not show this effect.
Correct Answer: Decreased fusion rate
2123. (4051) Q5-2705:
The upper-end vertebra for measurement of kyphosis should always be:
1) T1
3) T3
2) T2
5) The most inclined upper vertebra on the sagittal plane.
4) T4
As in measurement of any spinal curve, the upper-end reference is the most inclined upper vertebra. One should carefully select this level and ensure that comparison measurements utilize the same vertebral levels.
Correct Answer: The most inclined upper vertebra on the sagittal plane.
2124. (2277) Q5-2706:
Which of the following aspects of reconstruction of chronic posttraumatic dislocation of the radial head has not been shown to improve the success rate:
1) Apex-posterior angulation
3) Slight lengthening
2) Rigid fixation
5) Radiocapitellar pin fixation
4) Early range of motion
Reconstruction with an ulnar osteotomy with apex-posterior angulation and slight lengthening with rigid fixation to allow early range of motion have been shown to improve results. Radiocapitellar pin fixation may cause arthrosis, breakage, and infection so it is not routinely recommended.
Correct Answer: Radiocapitellar pin fixation
2125. (2278) Q5-2707:
In patients with idiopathic adolescent scoliosis, clinically noticeable shortness of breath from restrictive lung disease begins to occur at a mean curve of:
1) 45° thoracic
3) 55° thoracic
2) 45° lumbar
5) 95° thoracic
4) 75° thoracic
Clinically noticeable pulmonary compromise begins to occur at a mean curve of 75° thoracic, although a decrease can be measured by pulmonary function testing with curves as little as 50°. Cor pulmonale does not occur until a curve is larger then 100°.
Correct Answer: 75° thoracic
2126. (2279) Q5-2708:
Which of the following statements is true of patients with idiopathic scoliosis not treated surgically (long-term follow-up data) when compared to nonscoliotic controls:
1) Patients had an increased mortality rate.
3) Patients had an increased prevalence of back pain.
2) Patients had an increased rate of neuropathy.
5) Patients had an increased rate of spinal fractures.
4) Patientsâ curves did not worsen after adulthood.
Patients with idiopathic scoliosis have a mortality rate that is not statistically different from the general population. They do not have a significant risk of neuropathy or spine fracture. Their curves worsen in maturity if they are greater than approximately 40Â
°. They also have an increased prevalence of back pain.
Correct Answer: Patients had an increased prevalence of back pain.
2127. (2280) Q5-2709:
The cause of Ehlers-Danlos syndrome types I (gravis) and II (mitis) is a mutation in which of the following:
1) Fibrillin
3) Collagen type I
2) Fibroblast growth factor
5) Collagen type V
4) Collagen type II
Ehlers-Danlos syndrome is subclassified into at least nine types. Types I and II result from defects in type V collagen. Correct Answer: Collagen type V
2128. (4052) Q5-2710:
The most common neural injury after a supracondylar fracture of the distal humerus is:
1) Anterior interosseous nerve
3) Posterior interosseous nerve
2) Median nerve
5) Ulnar nerve
4) Radial nerve
The most commonly injured nerve after a supracondylar fracture of the distal humerus is the anterior interosseous nerve. An injury to this nerve results in the inability to flex the interphalangeal joint of the thumb and the distal interphalangeal joint of the index finger.
Correct Answer: Anterior interosseous nerve
2129. (2281) Q5-2711:
Which of the following conditions does not have a risk of cervical deformity greater than the general population:
1) Neurofibromatosis
3) Diastrophic dysplasia
2) Larsen syndrome
5) Down syndrome
4) Achondroplasia
Achondroplasia is associated with frequent stenosis of the foramen magnum in infancy, as well as lower cervical stenosis. However, it is not associated with an actual deformity of the cervical spine. By contrast, neurofibromatosis, Larsen syndrome, and diastrophic dysplasia are associated with infantile cervical kyphosis. Down syndrome is associated with the risk of upper cervical instability.
Correct Answer: Achondroplasia
2130. (2282) Q5-2712:
Which of the following iliac osteotomies provides the greatest freedom of mobilization of the acetabular segment:
1) Salter osteotomy
3) Steel osteotomy
2) Pemberton osteotomy
5) Ganz osteotomy
4) Chiari osteotomy
Osteotomies that are made closest to the acetabulum provide the greatest freedom of mobilization. Of the choices provided, the Ganz or Bernese osteotomy is made closest to the acetabulum.
Correct Answer: Ganz osteotomy
2131. (2283) Q5-2713:
Displaced tibial eminence fractures contain the attachment of which of the following structures in addition to the anterior cruciate ligament:
1) The lateral meniscus
3) The posterior cruciate ligament
2) The medial meniscus
5) The fat pad
4) The patellar ligament
Displaced tibial eminence fractures have been shown by arthroscopy to routinely contain the anterior attachment of the lateral meniscus. In addition, the anterior tibial attachment of the meniscus is torn. The cruciate and the meniscus pull the fragment in different directions.
Correct Answer: The lateral meniscus
2132. (2284) Q5-2714:
A 10-year-old boy sustains a type III avulsion of the anterior tibial eminence. When his knee is placed in extension, the fragment does not reduce. Which of the following factors is likely preventing its reduction:
1) Interposed anterior horn of medial meniscus
3) Interposed cartilage flap of tibial plateau
2) Interposed stump of anterior cruciate ligament
5) Opposing pull of cruciate and lateral meniscus
4) Increased intra-articular pressure from hematoma
Type III tibial eminence fractures usually contain attachments of both the anterior cruciate ligament and the lateral meniscus. The opposing pull of both of these structures often prevents reduction during extension of the knee.
Correct Answer: Opposing pull of cruciate and lateral meniscus
2133. (2285) Q5-2715:
A 13-year-old boy who underwent in situ fixation of slipped capital femoral epiphysis 1 year ago calls your office to complain of knee pain on the other side. He is able to bear his weight on the leg. You recommend:
1) No sports for 1 month and an office visit if the symptoms continue
3) Crutches and an office visit within 24 hours
2) An office visit within the next 2 weeks for evaluation
5) Arthroscopy of the knee
4) Magnetic resonance image of the knee and an office visit if the results are abnormal
This patient most likely has a contralateral slipped capital femoral epiphysis. It may even be in the âpreslipâ category. Acute progression to an unstable slip is possible at any time and may lead to avascular necrosis and permanent loss of motion.
Therefore, urgent examination with physical examination and plain radiographs is necessary. Correct Answer: Crutches and an office visit within 24 hours
2134. (2286) Q5-2716:
An 8-year-old boy suffers a hip dislocation while playing tackle football. His hip is reduced by closed means, and he is immobilized in abduction for a month. Radiographs at that time demonstrate a normal appearance of the hip. You recommend:
1) Discharge from follow-up
3) No sports and return to clinic in 2 months
2) Return to all sports and follow-up in 2 months
5) Femoral varus osteotomy
4) Hip abduction orthosis
This patient may suffer avascular necrosis as a result of the dislocation. Avascular necrosis may not become evident until 1 year after the accident, so continued surveillance and protection are recommended.
Correct Answer: No sports and return to clinic in 2 months
2135. (2287) Q5-2717:
Which population of patients with cerebral palsy is at greatest risk of neuromuscular hip subluxation:
1) Monoplegics
3) Spastic diplegics
2) Spastic hemiplegics
5) Athetoid
4) Total-involvement
Patients with total-involvement cerebral palsy are at the greatest risk of hip subluxation. The rate is documented to be between 25% and 60%.
Correct Answer: Total-involvement
2136. (2288) Q5-2718:
Which group of children with cerebral palsy are at greatest risk of hip subluxation after selective dorsal rhizotomy:
1) Under age 4
3) Spastic diplegics
2) Over age 10
5) Spastic hemiplegics
4) Nonambulators
Nonambulators with some degree of pre-existing hip migration are at highest risk of hip subluxation after selective dorsal rhizotomy. Patients over the age of 10 and those with hemiplegia are not typically offered selective rhizotomy.
Correct Answer: Nonambulators
2137. (2289) Q5-2719:
Which of the following muscles is most often preserved during adductor tenotomy for patients with cerebral palsy:
1) Adductor longus
3) Gracilis
2) Adductor brevis
5) Pectineus
4) Adductor magnus
Adductor tenotomy performed on patients with cerebral palsy typically involves multiple releases until abduction of more than 60Â
° is obtained. This usually begins with release of the longus, brevis, and gracilis (with the pectineus if necessary). The adductor magnus is almost never released because of its important extensor function.
Correct Answer: Adductor magnus
2138. (2290) Q5-2720:
The parameter most often recommended to follow the reciprocal relationship of the femoral head to the acetabulum in patients with cerebral palsy is known as the:
1) Migration index
3) Stulberg index
2) Epiphyseal extrusion index
5) Tonnis index
-
Acetabular index
The migration index (of Reimer) is most commonly used to track the femoral-acetabular relationship in patients with cerebral palsy because it most accurately portrays the progressive migration of the femoral head that may occur. The acetabular index only measures the acetabular response. The epiphyseal extrusion index is used for patients with Perthes disease (where the epiphysis deforms). The Stulberg index is for late outcome of Perthes, and the Tonnis index is for developmental dysplasia of the hip.
Correct Answer: Migration index
2139. (2291) Q5-2721:
The upper limit for a normal migration index in young children is less than:
1) 0%
3) 10%
2) 5%
-
35%
4) 25%
The migration index (of Reimer) is the percentage of the femoral head lateral to the Perkins line. The index is used to quantify hip migration in patients with cerebral palsy. It is more useful than the center-edge angle because it is a linear variable angle and because the center of the aspherical, immature femoral head may be hard to accurately identify. The upper limit of a normal migration in young children is listed as 25%.
Correct Answer: 25%
2140. (2292) Q5-2722:
The acetabular sourcil is best described as:
-
The lateral articular border
3) The acetabular angle
2) The teardrop
5) A cyst forming in hip dysplasia
4) A degenerative osteophyte
The acetabular sourcil is a lateral articular border, which normally should be downsloping and below the dome of the acetabulum. In dysplastic hips, the femoral head pushes the acetabular sourcil up and gives it an upsloping shape.
Correct Answer: The lateral articular border
2141. (2293) Q5-2723:
In patients with single thoracic idiopathic scoliosis treated with posterior pedicle screw constructs, the distal extent of the fusion may be stopped at which of the following levels with respect to the neutral vertebra and still routinely maintain balance:
1) Three levels above
3) One level above
2) Two levels above
5) Two levels below
4) One level below
In single thoracic idiopathic scoliosis, the distal extent of the fusion may be stopped at one level above the neutrally rotated vertebra and still maintain trunk balance.
Correct Answer: One level above
2142. (2294) Q5-2724:
The greatest number of malpractice suits in pediatric orthopedics relates to which diagnosis:
1) Fractures
3) Spinal deformities
2) Compartment syndromes
5) Congenital anomalies
4) Tumors
The largest number of malpractice claims in pediatric orthopedics relates to fractures. This diagnosis accounts for 77% of all claims, in contrast to 57% for adults. However, the mean amount paid for each claim is lower than other diagnostic groups. Orthopedic surgeons should be aware of the issue when taking care of children with fractures.
Correct Answer: Fractures
2143. (2295) Q5-2725:
For a given femur fracture pattern and age, which method of treatment causes the longest time to union in children ages 6 to 12 years:
1) Traction and cast
3) External fixation
2) Immediate hip spica
5) Rigid nails
4) Flexible nails
External fixation produces the longest times to healing, presumably because of load-shielding. The fracture ends may also be distracted.
Correct Answer: External fixation
2144. (2296) Q5-2726:
Fractures that have a high specificity for child abuse include all of the following except:
1) Corner fractures
3) Scapular fractures
2) Rib fractures
5) Spiral femur fractures
4) Sternal fractures
Even though a spiral pattern suggests a rotational force, spiral fractures of the femur are not specific for abuse. All of the other patterns are highly specific although not absolutely diagnostic.
Correct Answer: Spiral femur fractures
2145. (2297) Q5-2727:
The differential diagnosis of abuse in a child with a long bone fracture includes all of the following conditions except:
1) Myelodysplasia
3) Rickets
2) Congenital insensitivity to pain
5) Metaphyseal dysplasia
-
Copper deficiency
All of the diagnoses except metaphyseal dysplasia place patients at increased risk of fracture, and therefore could be confused with nonaccidental injury.
Correct Answer: Metaphyseal dysplasia
2146. (2298) Q5-2728:
Among children with nonossifying fibromas, a risk of pathologic fracture exists mainly when a lesion exceeds what minimum percentage of a boneâs diameter:
1) 10%
3) 33%
2) 25%
-
75%
4) 50%
In a series from the Mayo clinic, all pathologic fractures associated with nonossifying fibromas in the lower extremity occurred in lesions larger than 50% of the transverse diameter.
Correct Answer: 50%
2147. (2299) Q5-2729:
In a bowed tibia, the tensile stress during weightbearing is greatest at which point:
-
The apex on the convex side
3) The proximal joint surface
2) The apex on the concave side
5) There is never a tensile stress on a tibia.
4) The distal joint surface
The moment becomes tensile at the apex on the convex side of a tibial bow, in contrast to the normal compressive stress in a straight diaphysis. The moment is equal to load multiplied by distance from the axis of load.
Correct Answer: The apex on the convex side
2148. (2300) Q5-2730:
Which of the following medications has not been shown to increase the risk of osteopenia or osteoporosis:
1) Glucocorticoids
3) Phenytoin
2) Antacids containing aluminum
5) Pamidronate
4) Phenobarbital
Pamidronate, a bisphosphonate, increases bone density by inhibiting resorption. All of the medications listed as possible answers decrease bone density.
Correct Answer: Pamidronate
2149. (2301) Q5-2731:
A 7-year-old gymnast complains of pain in her medial midfoot. Radiographs reveal that the navicular is narrower and more sclerotic on this side than the other. The midfoot appears somewhat flattened. Your next step is to recommend:
1) Open biopsy
3) Percutaneous drilling
2) Electrical stimulation
5) Observation with rest as needed
4) Intravenous pamidronate
The condition described best resembles Kohlerâs disease, or avascular necrosis of the tarsal navicular. This disease has a natural history of spontaneous improvement with time, and biopsy is not needed. Rest improves symptoms, although it is probably not mandatory. Sometimes with severe pain, a cast may help to speed improvement of symptoms.
Correct Answer: Observation with rest as needed
2150. (2302) Q5-2732:
Which part of the body is removed last in a posterior vertebral column resected via a posterior approach:
1) The lamina
3) The anterior wall of the vertebral body
2) The pedicles
5) The transverse process
4) The posterior vertebral cortex
To protect and stabilize the neural elements, the posterior vertebral cortex is left intact to be resected last.Correct Answer: The posterior vertebral cortex
2151. (2303) Q5-2733:
![]() |
![]() |
![]() |
Slide 1 Slide 2 Slide 3
An 18-year-old man has chronic pain in his right thigh, which is foreshortened with an increased anterior bow (Slides). What is the most likely diagnosis:
1) Chronic osteomyelitis
3) Pagetâs disease
2) Osteogenesis imperfecta
5) Fibrous dysplasia
4) Multiple enchondromas
This patient has polyostotic fibrous dysplasia. The diffuse enlargement in width is atypical for osteogenesis imperfecta or infection. Pagetâs disease is associated with increased sclerosis. Enchondromas produce more localized changes in the bones than fibrous dysplasia.Correct Answer: Fibrous dysplasia
2152. (2304) Q5-2734:
![]() |
Slide 1
This infant was born with an anomaly of the left lower extremity (Slides). What is the most likely diagnosis:
1) Clubfoot
3) Fibular hemimelia
2) Proximal focal femoral deficiency
5) Congenital pseudarthrosis of the tibia
4) Tibial hemimelia
This infant has tibial hemimelia, which is a complete absence of the tibia with resultant inversion of the foot. The presence of an intact fibula causing focal lateral prominence, a shortened but complete femur, and absence of a visible tibia rule out a diagnosis of clubfoot, proximal focal femoral deficiency, fibular hemimelia, and congenital pseudarthrosis of the tibia.Correct Answer: Tibial hemimelia
2153. (2305) Q5-2735:
![]() |
Slide 1
This child with tibial hemimelia does not have a proximal tibial remnant (Slides). The preferred treatment is:
1) Knee disarticulation
3) Centralization of the fibula at the knee and ankle
2) Femoral-fibular fusion with Syme disarticulation
5) Rotationplasty
4) Syme disarticulation
If the proximal tibial remnant is absent, the prospect of a functionally useful extension at the knee is nil. Disarticulation at the knee is the best option for this patient.Correct Answer: Knee disarticulation
2154. (2306) Q5-2736:
![]() |
![]() |
Slide 1 Slide 2
A 6-year-old girl has had a limp with minimal pain for approximately 6 months. Her radiographs are presented (Slides). She has been afebrile and thriving otherwise. The best treatment method would be:
1) Observation
3) Curettage, bone graft, and supplemental plate fixation
2) Spica cast immobilization
5) Methylprednisolone acetate injection
4) Proximal femoral resection
This patientâs limp is caused by a benign bone lesion, most likely a unicameral bone cyst. Other less likely diagnoses include an aneurysmal bone cyst and fibrous dysplasia. Treatment involves curettage of the cystic lesions, bone grafting (allograft), and supplemental fixation because of the risk of fracture and recurrence.Correct Answer: Curettage, bone graft, and supplemental plate fixation
2155. (2307) Q5-2737:
![]() |
Slide 1
Which of the following Risser signs most accurately describes the patient presented (Slide):
1) Risser 1
3) Risser 3
2) Risser 2
5) Risser 5
4) Risser 4
Despite its biologic variability, the Risser sign is one of the most useful indicators of maturity used in the management of pediatric spine disorders. The Risser sign is a depiction of the progressive ossification and fusion of the iliac apophysis, which begins anterolaterally and finishes posteromedially. The initial ossification (Risser 1) begins just after the peak height velocity, after triradiate cartilage closure, and approximately at the time of menarche. The completion of ossification and fusion usually takes 1½ to 2 years in girls and 2 to 3 years in boys. As the iliac apophysis matures and Risser 4 is reached, the Risser sign becomes a sclerotic line that is âwhiterâ than the adjacent ilium. Risser 0 and Risser 5 may be difficult to distinguish except that the cranial border of the ilium in Risser 0 is wavy or âruffled,â while it is smooth and sclerotic in Risser 5.Correct Answer: Risser 4
2156. (2308) Q5-2738:
![]() |
Slide 1
A 15-year-old boy presents with a spinal curvature and no other known disorders (Slide). His curve is 105° and corrects to 70Â
° with traction. His neurologic examination is normal. The best treatment method would be:
1) Observation until medical problems develop
3) Anterior endoscopic fusion and instrumentation
2) Brace until maturity
5) Anterior and posterior fusion with posterior instrumentation
4) Anterior open fusion and instrumentation
A large, rigid curve like this requires an anterior release and fusion to obtain flexibility. The anterior release should be followed by a posterior fusion with instrumentation either on the same day or after a period of traction. Anterior instrumentation alone increases the risk of cut out and incomplete correction.Correct Answer: Anterior and posterior fusion with posterior instrumentation
2157. (2309) Q5-2739:
![]() |
Slide 1
Numerous braces are available for controlling spinal deformities in growing children. Which type of brace is presented here (Slide):
1) Boston brace
3) Milwaukee brace
2) Charleston brace
5) Providence brace
4) Wilmington brace
The Milwaukee brace is the first modern scoliosis brace to demonstrate efficacy and to achieve widespread use. The Milwaukee brace is a cervicothoracolumbar orthosis. All of the other braces listed are thoracolumbosacral orthoses.
The Boston brace is prefabricated and is selected from measurements taken of the patient. The Charleston brace is a night-only brace and bends the patient to reverse the curve.
The Wilmington brace is made of thermoplastic material for rapid, on-site fabrication.
The Providence brace is made using computer-assisted machining to help achieve maximal correction.
Correct Answer: Milwaukee brace
2158. (2310) Q5-2740:
![]() |
Slide 1
A patient has idiopathic scoliosis measuring 62° thoracic convex to the right and 40° lumbar convex to the left (Slide). The sagittal film shows a relatively decreased thoracic kyphosis and decreased lumbar lordosis. The patient is 13 years old and is 2 months postmenarchal. Which of the following treatments is most appropriate:
1) Milwaukee brace treatment
3) Posterior fusion from T4 to L3
2) Posterior fusion from T4 to L2
5) Posterior fusion from T4 to L5
4) Posterior fusion from T4 to L4
This curve may be classified as a King type II or III. The curve is too large for bracing. Only the thoracic curve must be fused. Because the stable vertebra is L2, the fusion does not need to go below this level. Other acceptable options include anterior thoracic instrumentation and fusion from T4 to T12 or posterior instrumented fusion to L1.Correct Answer: Posterior fusion from T4 to L2
2159. (2311) Q5-2741:
![]() |
Slide 1
Which of the following terms best describes the condition shown (Slide):
1) Calcaneovalgus
3) Posteromedial bow
2) Equinovarus
5) Metatarsus adductus
4) Fibular hemimelia
This patientâs foot is in calcaneovalgus. Treatment is not required because calcaneovalgus spontaneously resolves.Correct Answer: Calcaneovalgus
2160. (2312) Q5-2742:
![]() |
![]() |
Slide 1 Slide 2
A 4-year-old boy is brought to the doctor for evaluation of his feet. His mother is concerned that he has no arch. He does not have any pain in his feet, and there are no calluses on the plantar surface. Radiographs of one of the feet are shown below. You recommend:
1) Open reduction of the talocalcaneal joint
3) Calcaneal neck lengthening
2) Calcaneal slide osteotomy
5) Observation
4) Medial arch support
The radiographs demonstrate a normal appearance of a foot in a young child. The angle between the talus and calcaneus is increased at this age. The arch in a young child is not as well developed as in an older child. The navicular has not ossified yet. No treatment is needed.Correct Answer: Observation
2161. (2313) Q5-2743:
![]() |
![]() |
Slide 1 Slide 2
A 13-year-old girl is brought to the office for evaluation of her feet. During gait, she has a foot progression angle of 45° outward on each side. Her transmalleolar axis is 20° outward on each side. Her hips have internal rotation in extension of 40° and external rotation in extension of 40°. Her clinically estimated anteversion is 20°. She has popliteal angles of 20° bilaterally and full knee extension during swing. Standing radiographs of the feet are presented. Which factor is most likely to be the cause of her toeing-out:
1) Femoral retroversion
3) Dynamic knee torsion
2) External tibial torsion
5) Tight hamstrings
4) Forefoot abduction
This patient has significant forefoot abduction demonstrated on the standing foot radiographs. The femoral anteversion of 20° is within normal limits for age. The transmalleolar and popliteal angles are also within normal limits. There is no evidence of dynamic knee torsion as a factor in alignment of children.Correct Answer: Forefoot abduction
2162. (2314) Q5-2744:
![]() |
Slide 1
After being dragged by an automobile, a 7-year-old child sustained injuries to his foot (Slide). Following reconstructive surgery, he is most likely to be left with a permanent deficit in which of the following:
1) Ankle inversion
3) Ankle dorsiflexion
2) Ankle eversion
5) Growth of the navicular
4) Growth of the distal tibia
This patient sustained a dorsal degloving injury. The anterior tibialis tendon is gone, as are some of the toe dorsiflexors. The anterior tibial function is difficult to reconstruct, even after tendon transfer, because of scarring at the insertion site.Correct Answer: Ankle dorsiflexion
2163. (2315) Q5-2745:
![]() |
Slide 1
What best describes the anomalies depicted (Slide):
1) Incarcerated hemivertebra
3) Congenital diastematomyelia
2) Congenital hemivertebra
5) Block vertebra
4) Congenital hemivertebrae plus bar
The Slide shows two congenital hemivertebrae on the convex side plus a bar on the right side. An incarcerated hemivertebra fits into indentations in the adjacent vertebrae, causing no net deformity. A block vertebra is a pair of fully fused vertebrae usually without significant deformity.Correct Answer: Congenital hemivertebrae plus bar
2164. (2506) Q5-2977:
Compared to viral vectors, the advantages of non-viral vectors for gene delivery include all of the following except:
1) Safety
3) More efficiency
2) Less immunogenicity
5) Special packaging cell line
4) Easier production
Due to safety concerns, immunogenicity, and production issues associated with viral vectors, non-viral delivery systems were developed by complexing of genes (DNA) to various chemical formulations. This makes the DNA stable and increases its uptake. They include plasmids, peptides, cationic liposomes, DNA-ligand complexes (recognize specific cell-surface receptors, leading to receptor-mediated uptake), and gene gun (particles of gold coated with DNA, forced into the cells with high velocity bombardment). However, their efficiency is lower than viral vectors.Correct Answer: More efficiency
2165. (2507) Q5-2978:
All of the following have been used as viral vectors for gene delivery except:
1) Adeno-associated virus
3) Herpes simplex virus
2) Rotavirus
5) Retroviral vector
4) Lentivirus
A retroviral vector derived from the Moloney murine leukemia retrovirus is among the best-developed viral vectors. Other viral vectors include adenovirus, adeno-associated virus, and herpes simplex virus. Novel vector systems based on lentivirus, a type of retrovirus, which includes human immunodeficiency virus (HIV), are being developed.Correct Answer: Rotavirus
2166. (2508) Q5-2979:
The principle of homologous recombination in gene therapy is used to:
1) Replace a defective gene by a wild-type gene
3) Supplement a wild-type gene
2) Suppress the expression of a mutant gene
5) Replacement of a defective gene by a normal gene
4) Alter the expression of a mutant gene
Novel approaches to treating genetic diseases involve gene repair or replacement rather than gene supplementation. One such approach is based on homologous recombination (replacement of a defective gene by a normal gene).Correct Answer: Replacement of a defective gene by a normal gene
2167. (2509) Q5-2980:
The virus associated with most immune reactions is:
1) Adeno-associated virus
3) Adenovirus
2) âGuttedâ adenovirus
5) Herpes simplex virus
4) Retrovirus
Adenoviral vectors can cause inflammatory reaction due to immune activation, an event linked to the first death related to gene therapy. This occurred in September 1999 at the University of Pennsylvania in a clinical trial in which an 18-year-old patient received infusion of over a trillion adenoviral vectors directed to his liver, which triggered a systemic inflammatory response that became uncontrollable, leading to organ failure and death. Newer generation âguttedâ or âgutlessâ adenovirus vectors are nonimmunogenic.Correct Answer: Adenovirus
2168. (2510) Q5-2981:
Compared to ex vivo gene delivery system, the in vivo system is:
1) Technically complex
3) Safer
2) Target specific
5) More invasive
4) Less invasive
Two basic strategies exist for gene delivery. Direct, or in vivo, gene therapy involves direct introduction of vectors into the body. Indirect, or ex vivo, gene therapy involves removal of target cells from the body, vector introduction by incubation of the cells in vitro, and reimplantation. The in vivo system is less invasive.Correct Answer: Less invasive
2169. (2511) Q5-2982:
The gene that has been studied in greatest detail for application in osteoarthritis is:
1) p-53
3) Tissue inhibitors of metalloproteinases-4
2) Interleukin (IL)-13
5) BMP-2
4) IL-Receptor antagonist
Gene therapy has been suggested as a means of delivering sustained therapeutic levels of anti-arthritic gene products to diseased joints. Local gene delivery to the synovial tissue is preferred for osteoarthritis and other conditions affecting a few joints. It is less suited to address the extra-articular components of systemic conditions, such as rheumatoid arthritis. The gene that has been studied in greatest detail encodes the human IL-1 receptor antagonist (IL-Ra).Correct Answer: IL-Receptor antagonist
2170. (2512) Q5-2983:
The osteoinductive potential of LIM mineralization protein (LMP)-1 gene has been studied for clinical application in:
1) Fracture repair
3) Cartilage regeneration
2) Spinal fusion
5) Meniscal injury
4) Ligament healing
Identification of LIM mineralization protein-1 (LMP-1), a novel intracellular protein, is a step forward in osteoinductive proteins. Unlike BMP, which is a secreted protein that binds to cell-surface receptor to initiate a response, LMP-1 is an intracellular signaling molecule. Boden transfected bone marrow cells from rats ex vivo with LMP-1 gene using a DNA plasmid vector and used them during posterior thoracic and lumbar spine fusion in rats.Correct Answer: Spinal fusion
2171. (2513) Q5-2984:
The gene studied for application in osteoporosis and wear-induced osteolysis is:
1) Osteoprotegerin (OPG)
3) Transforming growth factor-b1
2) Bone morphogenetic protein (BMP)
5) IL-receptor antagonist
4) LMP-1
Various cytokines and cytokine antagonists hold promise as new therapeutic agents for osteoporosis. Baltzer et al showed that intramedullary injection of Ad-IL-1Ra gene in a murine ovariectomy model strongly reduced the loss of bone mass. Using a similar model, Bolon et al studied the effect of adenovirus-mediated transfer of osteoprotegerin, which showed more bone volume with reduced osteoclast numbers in axial and appendicular bones after 4 weeks compared to sham-operated mice.Correct Answer: Osteoprotegerin (OPG)
2172. (2514) Q5-2985:
Gene transfer to a cell using viral vectors is called:
1) Transduction
3) Transformation
2) Transfection
5) Augmentation
4) Conjugation
In vivo gene delivery involves the direct injection of vectors containing the genes into the body with the expectation that they will reach and transduce the target cell.Correct Answer: Transduction
2173. (2515) Q5-2986:
Which of the following genes has been shown to stimulate proteoglycan synthesis for prevention of disk degeneration:
1) LMP-1
3) Decorin
2) BMP-7
5) OPG
4) TGF-b1
Intervertebral disk degeneration has been associated with a progressive decrease in proteoglycan content of nucleus pulposus. The potential application of gene therapy for prevention of disk degeneration is to increase or maintain the proteoglycan content of nucleus pulposus. Thompson et al reported that addition of TGF-b1 to canine disk tissue in culture stimulated in vitro proteoglycan synthesis.Correct Answer: TGF-b1
2174. (2766) Q5-3264:
![]() |
Slide 1
This patient has numerous (> 6) café au lait spots (Slide). The most likely diagnosis is:
1) Fibrous dysplasia
3) Neurofibromatosis 1
2) Osteofibrous dysplasia
5) Osteogenesis imperfecta
4) Mazabraudâs syndrome
Neurofibromatosis is an autosomal dominant condition that occurs in a peripheral and central form. The peripheral type is more common and called neurofibromatosis 1 (NF1). The central type is characterized by bilateral acoustic neurofibromatosis (NF2). The clinical presentation is variable. Patients may either have a severe form of NF1 with hundreds of neurofibromas in the skin or they may have only café au lait spots and deep neurofibromas that are easily apparent.
Demographics of NF1 include:
-
Autosomal dominant with high degree of penetrance
-
Chromosomal abnormality - 17, NF1 gene (encodes neurofibromin)
-
Diagnostic criteria - two or more of the following: Six or more café au lait spots
Two or more neurofibromas of any type or one plexiform neurofibroma Freckling in the axillary or inguinal area
Optic glioma
Two or more Lisch nodules Bone lesion
First-degree relative with NF1
Correct Answer: Neurofibromatosis 1
2175. (2767) Q5-3265:
![]() |
Slide 1
This patient has numerous (> 6) café au lait spots (Slide). Which of the following describes the correct inheritance pattern:
1) Autosomal recessive
3) Autosomal dominant with high degree of penetrance
2) Autosomal dominant with low degree of penetrance
5) X-linked dominant
4) X-linked recessive
Neurofibromatosis is an autosomal dominant condition that occurs in a peripheral and central form. The peripheral type is more common and called neurofibromatosis 1 (NF1). The central type is characterized by bilateral acoustic neurofibromatosis (NF2). The clinical presentation is variable. Patients may either have a severe form of NF1 with hundreds of neurofibromas in the skin or they may have only café au lait spots and deep neurofibromas that are easily apparent.
Demographics of NF1 include:
-
Autosomal dominant with high degree of penetrance
-
Chromosomal abnormality - 17, NF1 gene (encodes neurofibromin)
-
Diagnostic criteria - two or more of the following: Six or more café au lait spots
Two or more neurofibromas of any type or one plexiform neurofibroma Freckling in the axillary or inguinal area
Optic glioma
Two or more Lisch nodules Bone lesion
First-degree relative with NF1
Correct Answer: Autosomal dominant with high degree of penetrance
2176. (2808) Q5-3309:
The bone graft used in the Grice procedure for the treatment of neurogenic valgus feet should be placed:
1) Between the cuboid and calcaneus
3) In the sinus tarsi parallel to the axis of the tibia
2) Among the metatarsal bones
5) None of the above
4) In the calcaneus after the osteotomy
In 1945, Grice developed the use of subtalar extra-articular arthrodesis as a treatment for paralytic valgus feet. Grice demonstrated that blocking the tarsal sinus opening prevents the calcaneus from rotating posterolaterally and thus blocks the heel of a foot from shifting into the valgus position. A bone block must be placed at such an angle in the tarsal sinus that when posterolateral migration of the calcaneus is attempted, the graft compresses. If a graft is placed too far forward, then the posterolateral movement of the calcaneus increases insertional distance between blocked areas, the graft collapses, and the foot goes into valgus.Correct Answer: In the sinus tarsi parallel to the axis of the tibia
2177. (2816) Q5-3317:
A patient with myelomeningocele has good quadriceps function, fair hamstring motion, and no active ankle movement. He has bilateral clubfoot. His motor level is:
1) L2
3) L4
2) L3
5) S1
4) L5
The hamstrings are innervated at a level just proximal to the anterior tibialis muscle (with the highest innervated ankle motor at L5). Therefore, the patientâs motor level is best estimated at L4.Correct Answer: L4
2178. (2817) Q5-3318:
In a type 1 Monteggiaâs fracture, the position of reduction is:
1) Flexion of the elbow and supination of the forearm
3) Extension of the elbow and pronation of the forearm
2) Flexion of the elbow and pronation of the forearm
5) Extension of the elbow and midposition of the forearm
-
Extension of the elbow and supination of the forearm
Type 1 Monteggiaâs fracture is associated with an anterior dislocation of the radial head and an apex anterior fracture of the ulna. Type 1 Monteggiaâs fracture is best treated by flexion and supination. The same maneuvers that reduce the ulna will also stabilize the radial head.Correct Answer: Flexion of the elbow and supination of the forearm
2179. (2818) Q5-3319:
What is the approximate risk of growth disturbance of distal radial Salter II fractures:
1) 50%
3) 15%
2) 25%
-
1%
4) 5%
Although distal radial physeal fractures are common, growth disturbance is rare with a 1% occurence. Growth disturbance poses a challenge for follow-up, but physicians should at least mention the risk to patients.Correct Answer: 1%
2180. (2819) Q5-3320:
Which of the following fracture situations would be most appropriate for internal fixation vs. attempted closed reduction:
-
A 7-year-old girl with 35° angulation of greenstick radius and ulna fracture
3) A 9-year-old girl with 30° angulation of complete radius and ulna fracture
2) A 7-year-old boy with 35° angulation of complete radius and ulna fracture
5) A 15-year-old girl with 25° angulation of complete radius and ulna fracture
4) A 12-year-old boy with 25° angulation of complete radius and ulna fracture
Remodeling potential is significantly decreased at skeletal maturity. The first four scenarios represent skeletally immature patients. The 15-year-old girl should be treated like an adult to achieve the most acceptable result. All other patients deserve an attempt at closed reduction.Correct Answer: A 15-year-old girl with 25° angulation of complete radius and ulna fracture
2181. (2820) Q5-3321:
All of the following may be signs and symptoms of a tethered cord except:
1) Back pain
3) Meryonâs sign
2) Leg pain
5) Increasing motor weakness
4) Increasing contracture
A tethered cord is common in patients with myelomeningocele. A tethered cord occurs due to adhesion of the dysplastic distal cord to its surrounding structures. The onset of a tethered cord may be exacerbated by closure of the open cord. Signs and symptoms of a tethered cord may include back pain, leg pain, increasing motor weakness, and increasing contracture. Alternate explanations for some of these signs and symptoms exist as well.
An adult with Meryon's sign may drop a child he or she is holding due to weakness of the pectorals and latissimus muscles. Meryon's sign is also common in patients with myelomeningocele who have a Chiari's malformation. In this case, the patient with myelomeningocele does not experience weakness of the pectorals and latissimus muscles due to a tethered cord. Meryon's sign also occurs in some myopathies.
Correct Answer: Meryonâs sign
2182. (2821) Q5-3322:
A 15-year-old patient with osteogenesis imperfecta (OI) is examined due to increasing knee flexion contractures and generalized muscle tightness of the lower extremities. The most likely explanation for these symptoms is:
1) Spondylolysis
3) Osteoarthritis associated with OI
2) Scoliosis
5) Disuse
4) Basilar invagination
Patients with OI have a significant risk of basilar invagination. In patients with OI the skull settles around the foramen magnum and cervical spine, causing the brainstem to become indented. Signs and symptoms include increased spasticity (described here as muscle tightness), contractures, hyperreflexia, clonus, respiratory depression, and changes in voice or swallowing. Basilar invagination is most common in type IV OI. Although osteoarthritis is more common in patients with OI than in the general population at a given age, the 15-year-old patient in this case is too young to develop osteoarthritis. Spondylolysis would not produce this constellation of symptoms unless associated with severe listhesis.Correct Answer: Basilar invagination
2183. (2822) Q5-3323:
A 6-year-old boy with osteogenesis imperfecta has had three fractures of his proximal femur resulting in progressive bowing, which has now reached 70°. He has pain in his thigh and a sclerotic stress line at the apex of the bow. The best treatment is:
1) Closed osteoclasis and spica cast
3) Osteotomy and plate fixation
2) Osteoclasis and external fixator
5) Observation
4) Osteotomy and internal fixation with an expandable rod
As bowing becomes greater, the likelihood of a re-fracture increases. The presence of a lucent sclerotic line and pain increases the risk of re-fracture. Intramedullary fixation is preferred to plate fixation because plate fixation increases the risk of stress risers. Expandable rods that bridge the entire bone from one epiphysis to the other are the best treatment for excessive bowing. Examples include the Bailey Dubow or the Fassier rods.Correct Answer: Osteotomy and internal fixation with an expandable rod
2184. (2823) Q5-3324:
A 10-year-old boy steps on a nail. The nail is promptly removed and inspected in the emergency department. No evidence of foreign material is found in the patientâs foot. One week later, the boy develops increased pain and swelling in his foot with inability to bear weight. Plain films show no abnormality. The appropriate next step of treatment would be:
1) Irrigation and debridement of the foot
3) Bone scan
2) Magnetic resonance imaging (MRI) of the foot
5) Repair of flexor tendon laceration
4) Computed tomography scan
Penetrating injuries to the sole of the foot commonly inoculate bacteria to the deep spaces of the foot. Pseudomonas, other gram-negative organisms, and Staphylococcus are examples of bacteria that commonly can become inoculated. An MRI would help to localize the site and stage of the infection. The MRI determines whether the surgeon should aspirate the infection and treat it with antibiotics or operatively debride the infection. Pseudomonas often requires debridement.Correct Answer: Magnetic resonance imaging (MRI) of the foot
2185. (2824) Q5-3325:
Which of the following is not a typical characteristic of Scheuermannâs kyphosis:
1) Endplate irregularity
3) Narrowing of disk space
2) Wedging of three vertebrae
5) Curve apex at or above T8
4) Schmorlâs nodes
Scheuermannâs kyphosis may affect any region of the thoracic spine; it also may affect the thoracolumbar junction or the lumbar spine. Endplate irregularity, wedging of three vertebrae, narrowing of disk space, and Schmorlâs nodes are characteristic of Scheuermannâs kyphosis.Correct Answer: Curve apex at or above T8
2186. (2825) Q5-3326:
A 17-year-old boy presents with thoracic kyphosis. He has minimal discomfort in his back. Radiographs show a 62° Scheuermannâs kyphosis, with wedging of T8-T10. His Risser sign is 4, and his Tanner stage is 4. He states that he is not cosmetically aware of his kyphosis. Recommended treatment includes:
1) A Milwaukee brace
3) A posterior spine fusion
2) A Boston brace
5) Observation
4) An anterior and posterior spine fusion
This patient is too skeletally mature to benefit from bracing. Bracing is not likely to change the natural history of the curve at maturity. Surgery is indicated if a patient experiences pain or dissatisfaction with the appearance of the back. Otherwise, the natural history is benign and observation is indicated.Correct Answer: Observation
2187. (2826) Q5-3327:
Adults with untreated Scheuermannâs kyphosis do not differ from controls in terms of this parameter:
1) Pulmonary function
3) Activity levels in job
2) Severity of back pain
5) Presence of scoliosis
4) Location of greatest backache
Patients with Scheuermannâs kyphosis have no decrease in pulmonary function compared to controls; however, they have increased severity of back pain, seek jobs with lower activity levels, and have more thoracic back pain and less trunk extension. They also have a 30% incidence of scoliosis of less than 35°.Correct Answer: Pulmonary function
2188. (2827) Q5-3328:
Correction of Scheuermannâs kyphosis from a posterior approach involves this mechanical principle:
1) Compression of the posterior column of the spine
3) Transverse approximation
2) Distraction of the posterior column
5) Hemiepiphyseodesis
4) Rod rotation
Compression of the posterior column is the principal method of correcting Scheuermannâs kyphosis. Compresssion is usually achieved by resecting portions of the elongation.Correct Answer: Compression of the posterior column of the spine
2189. (2828) Q5-3329:
A 13-year-old boy has a Scheuermannâs kyphosis of 68°, apex at T8. His Risser sign is 1. His mother is concerned about his appearance and possible future progression. Recommended treatment includes:
1) A Milwaukee brace
3) A posterior spine fusion
2) Exercise
5) Observation
4) An anterior and posterior spine fusion
Brace treatment for Scheuermannâs kyphosis is successful in decreasing the kyphosis, usually permanently, by 10° to 20° if worn properly. A patientâs Risser sign must be below 3. Because this patient has a Risser sign of 1, bracing is appropriate treatment. The Milwaukee brace is the most effective type of brace for kyphosis. However, the Boston brace is also somewhat effective in treating patients with Scheuermannâs kyphosis because it corrects the compensatory lordosis and stimulates active correction of the thoracic curve.Correct Answer: A Milwaukee brace
2190. (2829) Q5-3330:
![]() |
Slide 1
The mother of a 4-month-old boy brings him to a physician to be evaluated for a swollen leg (Slide). The most likely diagnosis is:
1) Rickets
3) Scurvy
2) Osteogenesis imperfecta (OI)
5) Caffeyâs disease
4) Nonaccidental injury
This radiograph shows two fractures in different stages of healing. Note the old femur fracture at the top of the field. No evidence of decreased cortical thickness, diaphyseal thinning, or bowing suggests OI. The physis of the distal femur and proximal femur show no signs of rickets. The presence of fractures rather than periosteal reaction make Caffeyâs disease unlikely. The fractures in scurvy are more commonly located in the physis. The diagnosis of nonaccidental injury should be made only after performing a thorough patient history and physical.Correct Answer: Nonaccidental injury
2191. (2830) Q5-3331:
Which of the following is not a contraindication to the repair of a spondylolytic defect:
1) Age older than 35 years
3) Disk degeneration at the same level
2) Slip greater than 25%
5) Minimal symptoms
4) Bilateral pars defect
Repair of a spondylolytic defect is usually a successful procedure for relieving symptoms. Repair is not indicated in patients older than 35 years of age. Repair should not be done if there is a significant slip over a grade 1 at most or if the patient has minimal, tolerable symptoms or has symptoms complicated by a degenerative disk at the same level.Correct Answer: Bilateral pars defect
2192. (2831) Q5-3332:
A 14-year-old boy has grade 1 isthmic L5-S1 spondylolisthesis. He has no back pain and wants to play football. The patient should be advised to:
1) Play football if his condition is treated by grafting and wiring
3) Play football if he wears a lumbosacral orthosis during all games
2) Play football if he has a L5-S1 fusion
5) Play football as desired
4) Not play football at all
Asymptomatic spondylolisthesis occurs in approximately 5% of the pediatric population. Slips of up to grade 1 develop in at least one third of patients with asymptomatic spondylolisthesis. Most patients with asymptomatic spondylolisthesis are minimally symptomatic or asymptomatic. The patient presented in this scenario may be allowed to play football as long as he understands that he may develop symptoms.Correct Answer: Play football as desired
2193. (2832) Q5-3333:
Most cases of L5 spondylolysis develop in patients by:
1) Birth
3) Age 6 years
2) Age 3 years
5) Age 12 years
4) Age 9 years
A study was conducted to monitor the development of L5 spondylolysis in patients from birth to 6 years of age. The study showed that no cases of L5 spondylolysis were present at birth in any of the patients. The majority of the patients developed L5 spondylolysis by the time they had reached 6 years of age. A 45-year follow-up study showed that none of the 30 patients had more than a 40% slip and that only 1 patient required fusion.Correct Answer: Age 6 years
2194. (2833) Q5-3334:
Patients with high-grade spondylolisthesis develop a vertical position of the sacrum in response to which condition:
1) Lumbar lordosis
3) Olisthetic scoliosis
2) Tight hamstrings
5) Lumbosacral kyphosis
4) Lumbar kyphosis
As spondylolisthesis progresses to a high grade, the fifth lumbar vertebra shifts forward into a kyphotic position termed a slip angle. The slip angle can reach a value of more than 30° and results in verticalization of a patientâs sacrum to decrease force on the involved vertebra.Correct Answer: Lumbar kyphosis
2195. (2834) Q5-3335:
Which of the following is not a feature of Klippel-Trénaunay-Weber syndrome:
1) Varicose veins
3) Increased length of the involved limb
2) Cutaneous nevus
5) Seizure disorder
4) Increased width of the involved limb
Klippel-Trénaunay-Weber syndrome is a constellation of varicose veins, cutaneous nevus, and an increase in the length or width of the involved limb. Seizure disorder is a feature of von Hippel- Lindau disease.Correct Answer: Seizure disorder
2196. (2835) Q5-3336:
Epiphyseal osteochondroma is also known as:
1) Mafucciâs syndrome
3) Larsen-Johansson disease
2) Freibergâs fracture
5) Iselinâs disease
4) Trevorâs disease
Epiphyseal osteochondroma is also known as Trevorâs disease. Epiphyseal osteochondroma is localized to a specific region of the body, unlike multiple osteochondroma, which affects the entire body. Epiphyseal osteochondroma and multiple osteochondroma are unrelated disorders.Correct Answer: Trevorâs disease
2197. (2836) Q5-3337:
All of the disorders listed below are examples of osteochondrosis except:
1) Legg-Calvé-Perthes disease
3) Osgood Schlatter disorder
2) Scheuermannâs disease
5) Blount disease
4) Gorhamâs disease
The term osteochondrosis refers to symptomatic disorders involving cartilage growth. Cartilage affected by osteochondrosis may be epiphyseal, physeal, or apophyseal. Gorhamâs disease is not an example of osteochondrosis. Gorhamâs disease is a lymphatic disorder known as disappearing bone disease.Correct Answer: Gorhamâs disease
2198. (2837) Q5-3338:
Which of the following symptoms is not characteristic of congenital constriction band syndrome:
1) Circular grooves in the limb
3) Terminal amputations
2) Complex complete syndactyly
5) Lymphedema
4) Clubfoot
Syndactyly in constriction band syndrome is a fenestrated syndactyly, is incomplete, and has slits between the digits proximally. Syndactyly in constriction band syndrome is also not complex because the bones are not fused.Correct Answer: Complex complete syndactyly
2199. (2838) Q5-3339:
Which of the following conditions requires an ultrasound of the abdomen:
1) Russell-Silver syndrome
3) Idiopathic hemihypertrophy
2) Camurati-Engelmann disease
5) Blue rubber bleb nevus syndrome
4) Congenital constriction band syndrome
An ultrasound of the abdomen is indicated for patients with idiopathic hemihypertrophy to determine the development of Wilmsâ tumor. Children with idiopathic hemihypertrophy have an increased incidence of Wilmsâ tumor. Children require periodic monitoring, and they should have an abdominal ultrasound 2 to 3 times per year until 8 years of age.Correct Answer: Idiopathic hemihypertrophy
2200. (2839) Q5-3340:
How often should patients with hemihypertrophy have an abdominal ultrasound:
1) Once at 2 years old
3) Two to three times per year until 8 years old
2) Yearly until 5 years old
5) Yearly until 18 years old
4) Quarterly until 18 years old
Patients with idiopathic hemihypertrophy require periodic monitoring to determine the risk of Wilmsâ tumor. The best method to monitor patients with idiopathic hemihypertrophy is a renal ultrasound two to three times per year until the patient is 8 years of age.Correct Answer: Two to three times per year until 8 years old
2201. (2840) Q5-3341:
Which of the following rays is most commonly used to treat patients with macrodactyly of the foot:
1) First
3) Third
2) Second
5) Fifth
4) Fourth
The second ray is most commonly used to treat patients with macrodactyly of the foot. The third ray is the next most commonly used ray in treating patients with macrodactyly of the foot.Correct Answer: Second
2202. (2841) Q5-3342:
Macrodactyly of the foot commonly displays which of the following patterns of overgrowth:
1) Greatest dorsally and distally
3) Greatest plantarly and proximally
2) Greatest plantarly and distally
5) Greatest medially and proximally
4) Greatest laterally and distally
Macrodactyly displays overgrowth that is greatest plantarly and distally. Plantar overgrowth causes the sole to become convex plantarly and the toes to become dorsiflexed.Correct Answer: Greatest plantarly and distally
2203. (2842) Q5-3343:
A 1-year-old patient presents with a pseudarthrosis of the left clavicle. Which of the following conditions is most likely to coexist with pseudarthrosis of the left clavicle:
1) Neurofibromatosis
3) Osteogenesis imperfecta
2) Proteus syndrome
5) Dextrocardia
4) Fibrous dysplasia
Almost all cases of congenital pseudarthrosis involve the right clavicle. Cases of congenital pseudarthrosis involving the left clavicle often coexist with dextrocardia. Patients with pseudarthrosis of the clavicle respond well to standard orthopedic treatment, unlike patients with congenital pseudarthrosis of the tibia.Correct Answer: Dextrocardia
2204. (2843) Q5-3344:
A 14-year-old girl presents with a swollen foot. She had sustained a contusion to the dorsum of her foot for 4 weeks. Her temperature is 99.2° F. Her foot is tender to touch. She is able to dorsiflex and plantarflex her toes. Radiographs are normal. The best treatment is:
1) Physical therapy
3) Sciatic block
2) Ankle block
5) Blood culture
4) Ankle joint aspiration
This patient demonstrates signs of early reflex sympathetic dystrophy. She may later develop osteopenia, a positive bone scan, and contracture. Physical therapy is the best treatment for reflex sympathetic dystrophy in this case because it will increase the patientâs range of movement and her tolerance to touch. Multimodality therapy is also required to treat reflex sympathetic dystrophy. Analgesics may be appropriate treatment for the patient and can be supplemented by antidepressants, if needed.
Repeated regional block treatment is recommended for patients with established cases of reflex sympathetic dystrophy.Correct Answer: Physical therapy
2205. (2844) Q5-3345:
![]() |
Slide 1
A 10-year-old girl injured her neck (Slide) after falling and hitting her head. Recommended treatment includes:
1) Anterior fusion C 6-T1 with plate fixation
3) Anterior and posterior fusion C 6-T1 with plate fixation
2) Posterior fusion C 6-T1 with plate fixation
5) Cervicothoracic orthosis
4) Halo vest immobilization
The patient has an anterior compression fracture of C 7, which can be treated by cervicothoracic orthosis. Halo vest immobilization improves control of the head and upper cervical spine but is unnecessary treatment in this patientâs case. Although an anterior operation is an optional treatment for an anterior compression fracture, it is also unnecessary in this patientâs case.Correct Answer: Cervicothoracic orthosis
2206. (2845) Q5-3346:
![]() |
Slide 1
Which of the following conditions is represented in the radiograph (Slide):
1) Spina bifida of the first lumbar vertebra
3) Burst fracture
2) Diastematomyelia
5) Flexion-rotation injury
4) Chance fracture
This radiograph demonstrates a chance fracture. The posterior elements of the spine are separated, as indicated by the transverse fracture of the transverse processes, pedicles, and lamina. There is no significant element of rotation of the spine.Correct Answer: Chance fracture
2207. (2846) Q5-3347:
![]() |
Slide 1
A 15-year-old girl experienced an injury of L1 during a sledding accident (Slide). Her neurologic examination is normal. The best treatment is:
1) Thoracolumbosacral orthosis
3) Reduction and posterior fusion of T12-L1 with pedicle screws
2) Hyperextension cast
5) Reduction and posterior fusion of T12-L2 with pedicle screws
4) Corpectomy of T12 and anteroposterior instrumented fusion
This patient has a Chance fracture of L1, which is indicated by the compression of the anterior portion of L1 and the distraction of the posterior portion of L1. A hyperextension cast will not treat the patientâs Chance fracture completely. Operative treatment is needed to remove residual kyphosis at the thoracolumbar junction. Posterior column compression will also likely reduce the presence of residual kyphosis. Posterior fusion of T12-L2 with pedicle screws results in the best angular correction for a patient with a Chance fracture of L1. T12-L1 with hooks is an optional treatment for patients with a Chance fracture of L1 because residual kyphosis may persist even if the inferior lamina of L1 is intact.Correct Answer: Reduction and posterior fusion of T12-L2 with pedicle screws
2208. (2847) Q5-3348:
![]() |
Slide 1
Which of the following conditions is represented in this radiograph (Slide) of a childâs neck:
1) Osteoid osteoma of C 1
3) Rotatory subluxation
2) Jeffersonâs fracture
5) Os odontoideum
4) Hangman fracture
This radiograph indicates a rotatory subluxation of a childâs neck. Alteration of the cortical ring of the anterior atlas is present and superimposed on the lateral mass of atlas. Increased distance between the anterior atlas and axis is the result of rotation.Correct Answer: Rotatory subluxation
2209. (2848) Q5-3349:
![]() |
Slide 1
Which of the following conditions is represented by this computed tomography scan of a cervical spine (Slide):
1) Jeffersonâs fracture
3) Odontoid fracture
2) Osteoid osteoma
5) Chiariâs malformation
4) Rotatory subluxation
Partial superimposition of C 1 on C 2 is present in this radiograph. C 2 is the portion in the center of the frame. C 1 and C 2 would normally be on different cuts of a scan but are superimposed because the atlas overlaps the axis as it falls forward. If an imaginary line is drawn between the vertebral foramena of C 1 and across the body of C 2, an angle will form representing the degree of malrotation, which is approximately 30°.Correct Answer: Rotatory subluxation
2210. (2849) Q5-3350:
Which of the following conditions is not commonly associated with Chance fractures in children:
1) Renal trauma
3) Dural tear
2) Intestinal contusion
5) Multilevel spine trauma
4) Neurologic injury
Chance fractures in children are often associated with blunt contusions to abdominal structures. Blunt contusions to abdominal structures cause renal and intestinal trauma and distraction of the spinal cord and may be associated with dural tearing or neurologic injury. Neurologic injuries occur in fewer than half of children with Chance fractures that have blunt contusions to abdominal structures. Although spinal fractures caused by falls or axial loads can result in multilevel spinal injuries, multilevel spinal injuries do not commonly result from Chance fractures.Correct Answer: Multilevel spine trauma
2211. (2850) Q5-3351:
![]() |
Slide 1
The lesion indicated in this computed tomography (Slide) most likely represents:
1) Brodieâs abscess
3) Osteosarcoma
2) Eosinophilic granuloma
5) Osteoma
4) Osteoid osteoma
This patient has an osteoid osteoma with pain in the lumbar spine where the lesion is located. The patientâs osteoid osteoma shows hot on a photon emission computed tomography scan and is relieved by nonsteroidal anti-inflammatory agents. Most patients with osteoid osteoma with lesions have them in the posterior region of their vertebrae.Correct Answer: Osteoid osteoma
2212. (2851) Q5-3352:
![]() |
Slide 1
Which of the following conditions is represented by this radiograph (Slide) of an infantâs elbow:
1) Salter I fracture of distal humerus
3) Type 2 Monteggiaâs fracture dislocation
2) Elbow dislocation
5) Proximal radioulnar synostosis
4) Septic elbow
This patient has a proximal radioulnar synostosis. Patients with proximal radioulnar synostosis often have dysplasia of the proximal ulna, maldirection of the proximal radius, and cortical continuity of the ulna and radius.Correct Answer: Proximal radioulnar synostosis
2213. (2852) Q5-3353:
![]() |
Slide 1
Which of the following conditions is represented by this radiograph (Slide):
1) Developmental dysplasia of the hip
3) Hemophilic arthropathy
2) Juvenile rheumatoid arthritis
5) Rickets
4) Legg-Calvé-Perthes disease
This patient has early fragmentation, epiphyseal flattening, metaphyseal lucency and widening, and sclerosis typical of patients with Legg-Calvé-Perthes disease in the early stages.
In patients with juvenile rheumatoid arthritis, osteopenia and joint space narrowing are present, whereas in patients with developmental dysplasia, the acetabulum is shallow.
![]() |
Correct Answer: Legg-Calvé-Perthes disease 2214. (2853) Q5-3354:
Slide 1
Which of the following designations best describes this radiograph (Slide):
1) Herring antibody
3) Stulberg 4
2) Herring body
5) 30% epiphyseal extrusion
4) Catterall 1
This patient has Herring body involvement of the hip. As a result, this patient has less than a 50% loss of lateral column height and is in the early fragmentation phase. This patient has a Catterall involvement of at least 3. The Stulberg classification is used to rate a patientâs hip after it has healed.Correct Answer: Herring body
2215. (2854) Q5-3355:
![]() |
Slide 1
In which stage of Legg-Calvé-Perthes disease is this patient:
1) Initial
3) Reossification
2) Fragmentation
5) Degenerative
4) Healed
This patient is in the end of the fragmentation phase of Legg-Calvé-Perthes disease. Note the extreme flattening of the head and the lateral fragmentation.Correct Answer: Fragmentation
2216. (2855) Q5-3356:
![]() |
Slide 1
All of the following conditions are indicated by this radiograph (Slide) of a patient with Legg- Calvé-Perthes disease except:
1) Coxa magna
3) Trochanteric overgowth
2) Coxa brevis
5) Coxa plana
4) Coxa vara
The hip of this patient with Legg-Calvé-Perthes disease has undergone epiphyseal flattening and premature (primarily lateral) physeal arrest. The femoral head has regrown with an enlarged width. The radiograph shows elements of coxa magna, coxa brevis, trochanteric overgrowth, and coxa plana. Coxa vara is not present.Correct Answer: Coxa vara
2217. (2856) Q5-3357:
A 5-year-old girl is evaluated for Legg-Calvé-Perthes disease. She is in the fragmentation phase, and rotation is 10° internal and 15° external. Her epiphyseal extrusion index is 15%. Recommended treatment includes:
1) Femoral osteotomy
3) Traction
2) Iliac osteotomy
5) Observation
-
Shelf procedure
The patientâs restricted range of motion is normal for a patient in the fragmentation phase of Legg-Calvé-Perthe disease. The patientâs degree of epiphyseal extrusion is not extreme. Observation is the most appropriate course of treatment.Correct Answer: Observation
2218. (2857) Q5-3358:
What is the risk of hip arthroplasty for 50-year-old patients with Legg-Calvé-Perthes disease:
1) 10%
3) 33%
2) 25%
-
75%
4) 50%
Long-term study has shown that 50% of 50-year-old patients with Legg-Calvé-Perthes disease will have degenerative changes in their hips significant enough to make them candidates for hip arthroplasty.Correct Answer: 50%
2219. (2858) Q5-3359:
Which of the following descriptions reflects all transverse-plane factors affecting the position of the patientâs foot during gait:
-
Thigh-foot angle
3) Foot progression angle
2) Transmalleolar axis
5) Femoral anteversion
4) Heel bisector
The foot-progression angle is a reflection of all transverse-plane factors affecting the position of the patientâs foot during gait.Correct Answer: Foot progression angle
2220. (2859) Q5-3360:
In which of the following conditions is a Trendelenburgâs gait unlikely to occur:
1) L4 myelomningocele
3) Untreated developmental dysplasia of the hip
2) Legg-Calvé-Perthes disease
5) Osgood-Schlatter disease
4) Polio- L5
Patients with complications that cause pain around their hips or that decrease the abductor strength of their hips may develop a Trendelenburgâs gait.Correct Answer: Osgood-Schlatter disease
2221. (2860) Q5-3361:
In which muscle does Elyâs test detect spasticity or contracture:
1) Biceps femoris
3) Gastrocnemius
2) Rectus femoris
5) Semimembranosus
4) Soleus
Elyâs test is performed while the patient is prone and his or her hip is at maximal extension. Elyâs test is positive if flexion of the patientâs knee causes the patientâs hip to flex. A positive Elyâs test indicates spasticity or contracture of the rectus femoris.Correct Answer: Rectus femoris
2222. (2861) Q5-3362:
Staheliâs test is designed to detect hip flexion contracture. Which of the following tests also detects hip flexion contracture:
1) Silfverskiöldâs test
3) Thomas test
2) Oberâs test
5) Trendelenburgâs test
4) Jackâs test
Staheliâs test and the Thomas test are designed to detect hip flexion contracture. Staheliâs test is performed with a patient in the prone position. The lumbar lordosis is eliminated by flexing a patientâs hips, and then gradually extending the affected hip. The Thomas test is performed with a patient in the supine position. The lumbar lordosis is minimized by flexing a patientâs hips forward, allowing the affected hip to go into extension.Correct Answer: Thomas test
2223. (2862) Q5-3363:
Which of the following conditions has the highest rate of bilaterality:
1) Slipped capital femoral epiphysis
3) Developmental dysplasia of the hip
2) Legg-Calvé-Perthes disease
5) Fibular hemimelia
4) Discoid lateral meniscus
Slipped capital femoral epiphysis has a bilaterality rate of 25% to 40%. Legg-Calvé-Perthes disease, developmental dysplasia of the hip, discoid lateral meniscus, and fibular hemimelia have bilateral rates of less than 20%.Correct Answer: Slipped capital femoral epiphysis
2224. (2863) Q5-3364:
In which pattern does the most common presentation of Trevorâs disease occur:
1) Generalized involvement of all extremities
3) Proximal lateral areas of both lower extremities
2) Distal medial areas of one lower extremity
5) Proximal lateral areas of one upper extremity
4) Distal medial areas of both upper extremities
Trevorâs disease, also known as dysplasia epiphysialis hemimelia, refers to epiphyseal osteochondromas. Epiphyseal osteochondromas are usually few in number and are more common on the distal and medial portions of the epiphysis of one lower extremity.Correct Answer: Distal medial areas of one lower extremity
2225. (2864) Q5-3365:
At which age do patients most commonly present with dysplasia epiphysialis hemimelia (DEH):
1) Birth
3) Early adulthood
2) Between early childhood and early teen years
5) Older than 60 years
4) Middle age
Most cases of DEH occur in patients between early childhood and early teen years. Dysplasia epiphysialis hemimelia does not present at birth.Correct Answer: Between early childhood and early teen years
2226. (2865) Q5-3366:
![]() |
Slide 1
This radiograph (Slide) shows an 11-month-old girl with a Tonnis 3 developmental dislocation of the hip. Recommended treatment includes:
1) Pavlik harness
3) Open reduction and spica cast application
2) Closed reduction and spica cast application
5) Open reduction with Salter osteotomy
4) Open reduction with femoral shortening derotation osteotomy
Closed reduction and spica cast application is the best treatment for this patient with a Tonnis 3 developmental dislocation of the hip. Open reduction and spica cast application introduces additional risks of infection and vascular compromise and should not be performed unless closed reduction and spica cast application fails. This patient is too old to be controlled by a Pavlik harness.Correct Answer: Closed reduction and spica cast application
2227. (2866) Q5-3367:
The mechanical axis of the lower extremity is defined as the angle formed by lines drawn from the center of the hip to the center of the knee to the center of the ankle. The resulting value should be:
1) 15°
3) 7°
2) 10°
5) 0°
4) 5°
The mechanical axis should be 0°. Alignment following correctional osteotomies and arthroplasties must re-establish the mechanical axis.Correct Answer: 0°
2228. (2867) Q5-3368:
A 2-year-old boy presents with intoeing. An examination shows that his feet form an angle of 20° inward with the direction he is walking. Which of the following parameters describes his condition:
1) Thigh-foot angle
3) Angle of femoral anteversion
2) Tibial torsion angle
5) Transmalleolar axis
4) Foot progression angle
The 20° inward angle formed with the direction the patient is walking is a foot progression angle. This angle encompasses all factors influencing position and progression of the lower extremity during gait.Correct Answer: Foot progression angle
2229. (2868) Q5-3369:
Which of the following measurements reflects the mean lateral distal femoral joint angle with respect to the mechanical axis:
1) 90°
3) 84°
2) 87°
5) 78°
4) 81°
The lateral distal femoral angle with respect to the mechanical axis is 87°.Correct Answer: 87°
2230. (2869) Q5-3370:
A 2-year-old patient presents with bowed legs. The metaphyseal diaphyseal angle is 17°, and the mechanical axis shows 15° of varus bilaterally. The varus appears to be in the proximal tibia. No evidence of metabolic disease or dysplasia is present.
Recommended treatment inlcudes:
1) Observation
3) High tibial osteotomy
2) Knee-ankle-foot orthosis
5) Percutaneous hemiepiphysiodesis with drill
4) Medial staple hemiepiphysiodesis
This patient has infantile Blount disease. Brace treatment is appropriate to try, although it is not always successful. The best treatment is for the patient to wear a knee-ankle-foot brace full-time. High tibial osteotomy, medial staple hemiepiphysiodesis, and percutaneous hemiepiphysiodesis are inappropriate treatments for this patient at 2 years old. If the varus does not improve by the time the patient is 3.5 years old to 4 years old, then high tibial osteotomy should be performed.Correct Answer: Knee-ankle-foot orthosis
2231. (2870) Q5-3371:
A newborn infant presents with a knee complication. The patientâs knee hyperextends to 30° and flexes to 30°. The neurovascular examination is normal, and the patientâs hips are stable. No other skeletal complications are found. Radiographs show a line along the axis of the tibia intersecting the ossific nucleus of the distal femur, signaling significant hyperextension.
Recommended treatment inlcudes:
1) Observation
3) Pavlik harness
2) Serial casting
5) Open reduction of the knee with V-Y plasty
4) Prone skin traction to gradually flex the leg
This patient has a hyperextensible, but not dislocated, knee. In the radiograph, the line along the axis of the tibia would intersect the femur anterior to the ossific nucleus if the patientâs knee were dislocated. Hyperextensible knee is an in utero complication and resolves without treatment.Correct Answer: Observation
2232. (2871) Q5-3372:
A 12-year-old girl has genu valgum and requests correction. Radiographs reveal 12° valgus of the mechanical axis, with 2° arising in the distal femur and 3° arising in the proximal tibia. No evidence of other disorders are present. Recommended treatment includes:
1) Observation
3) Medial distal femur staple hemiepiphysiodesis
2) Knee-ankle-foot orthosis worn at night
5) Medial closing wedge osteotomy of the proximal tibia
4) Lateral opening wedge osteotomy of the distal femur
This patient has a significant amount of valgus. Valgus at the knee is evident when signaled by even a low number of degrees. The patient is at an age when medial distal femur staple hemiepiphysiodesis would be the best treatment for genu valgum. Medial distal femur staple hemiepiphysiodesis is a safe and effective procedure and is performed using small incisions, which allow for immediate ambulation.Correct Answer: Medial distal femur staple hemiepiphysiodesis
2233. (2872) Q5-3373:
Which of the following conditions is not part of the differential diagnosis of a valgus knee in a 4-year-old child:
1) Physiologic valgus
3) Multiple exostoses
2) Prior proximal metaphyseal fracture
5) Infantile Blount disease
4) Chondroectodermal dysplasia
Physiologic valgus, prior proximal metaphyseal fracture, multiple exostoses, and chondroectodermal dysplasia produce valgus. Infantile Blount disease produces varus.Correct Answer: Infantile Blount disease
2234. (2873) Q5-3374:
Which of the following methods of treatment has the lowest success rate in treating patients with congenital pseudarthrosis of the tibia:
1) Electromagnetic stimulation
3) Ilizarovâs method
2) Vascularized fibula graft
5) Ilizarovâs method over an intramuscular rod
4) Intramedullary (IM) rod fixation and bone graft
Electromagnetic stimulation has the lowest success rate in treating patients with congenital pseudarthrosis of the tibia. Vascularized fibula graft, Ilizarovâs method, IM rod fixation and bone graft, and Ilizarovâs method over an intramuscular rod have similar success rates.Correct Answer: Electromagnetic stimulation
2235. (2874) Q5-3375:
Which of the following methods of measuring limb length inequality includes the greatest number of factors leading to pelvic height difference:
1) Scanogram
3) Tape method
2) Orthoroentgenogram
5) Teleroentgenogram
4) Block method
The block method consists of adding height to the short limb in blocks until the pelvis is level. The block method measures differences in foot height and pelvic size as well as inequalities of the long bones. Because the teleroentgenogram shows the whole limb in one exposure, the exposure is susceptible to be parallax and distorted at the ends. The orthoroentgenogram and scanogram measure limb length inequality similarly, but the scanogram captures only the joints and eliminates the diaphyses.Correct Answer: Block method
2236. (2875) Q5-3376:
A patient with myelomeningocele is a community ambulator. She has a minimal Trendelenburgâs sign but has a calcaneus gait. Her motor level is:
1) L2
3) L4
2) L3
5) S1
4) L5
A calcaneus gait describes heel strike with no pushoff. This patient has active anterior tibialis with no gastrocsoleus. The Trendelenburgâs test shows that her gluteals are minimal at less than L4. The patientâs motor level is L5.Correct Answer: L5
2237. (2876) Q5-3377:
An 8-year-old child with spina bifida has a focal kyphosis measuring 100° with an apex at the first lumbar vertebra and a short trunk. The patientâs family is concerned about the childâs risk of skin breakdown posteriorly. Recommended treatment includes:
1) Milwaukee brace
3) Posterior fusion in situ to produce correction with growth
2) Correction in a hyperextension cast followed by a Milwaukee brace
5) Posterior vertebral excision and instrumentation
4) Anterior strut grafting
This patient has myelokyphosis, which occurs in some patients with a thoracic level of spina bifida. The myelokyphosis is caused by a lack of posterior spinal osteoligamentous elements and denervated musculature and has a sharp, single apex with a compensatory lordosis above and below the apex. Myelokyphosis is steadily progressive with growth. Bracing and casting are ineffective treatments for patients with myelokyphosis due to the focality of the complication and insufficient skin coverage over the apex. Posterior fusion in situ is mechanically ineffective in controlling the large focal curve occurring in myelokyphosis.
Anterior strut grafting in a growing child functions as an anterior bar that exacerbates the patientâs myelokyphosis. A kyphectomy is the best treatment for a patient with myelokyphosis. A kyphectomy entails a posterior resection of the apical vertebra and posterior instrumentation. Posterior instrumentation corrects the cantilever. A kyphectomy does not entail fusion beyond the resected area because such a fusion impedes growth.Correct Answer: Posterior vertebral excision and instrumentation
2238. (2877) Q5-3378:
![]() |
Slide 1
This is a radiograph (Slide) of a patient with myelomeningocele. At which of the following levels is the lesion located:
1) L1
3) L4
2) L2
5) S1
4) L5
This patient has active quadriceps (which are innervated through L2-L4) and adductors (which are innervated through L1-L3). Because the patientâs knees are slightly hyperextended there is no hamstring function. The patientâs right foot has some dorsiflexion. The lesion is rated as L4. However, the patientâs right side may be rated as L3.Correct Answer: L4
2239. (2878) Q5-3379:
![]() |
![]() |
Slide 1 Slide 2
A 1-year-old patient with L4 myelomeningocele presents with a foot complication. Radiographs are shown in neutral plantarflexion (Slide 1) and in maximal plantarflexion (Slide 2). Recommended treatment includes:
1) Ponsetiâs cast treatment
3) Complete subtalar release
2) Orthotic treatment
5) Excision of midtarsal bone
4) Open reduction of congenital talonavicular dislocation
This patient has a congenital vertical talus, also known as a congenital dorsolateral dislocation of the talonavicular joint. Because the navicular is not yet ossified, the dorsal position of the first metatarsal line illustrates evidence of the patientâs congenital vertical talus when compared to the position of the talus. The congenital vertical talus is fixed because it does not become reduced upon maximum plantarflexion. The best treatment for congenital vertical talus is open reduction of the congenital talonavicular dislocation, with tendon lengthening.Correct Answer: Open reduction of congenital talonavicular dislocation
2240. (2879) Q5-3380:
![]() |
Slide 1
An 8-year-old girl with myelomeningocele has sustained warmth and swelling of her leg for 2 weeks. She does not recall any trauma. She has had a temperature of 101° on several occasions. Her radiograph (Slide) is shown below. The most likely diagnosis is:
1) Osteomyelitis of the tibia
3) Deep venous thrombosis
2) Fracture of the distal tibia
5) Osteogenic sarcoma
4) Ewingâs tumor
This patient has a Salter 1 physeal fracture of the distal tibia, which was probably caused by stress that remained unrecognized due to the patientâs lack of pain. Patients with spina bifida experience extensive periosteal reaction because they do not get early immobilization.Correct Answer: Fracture of the distal tibia
2241. (2880) Q5-3381:
![]() |
Slide 1
This radiograph (Slide) shows a 5-year-old boy with an L4 myelomeningocele. He can ambulate with a walker. Recommended treatment includes:
1) Observation
3) Bilateral adductor tenotomy
2) Abduction bracing
5) Bilateral adductor tenotomy, femoral and iliac osteotomy
4) Bilateral adductor tenotomy and femoral osteotomy
This patient has symmetrical high, longstanding dislocations. Because his level is L4, he has no abductor function. The patient has multiple contraindications to surgery, including current symmetry, lack of abduction power, young age, and an inability to walk without a walker. His ability to walk would likely be hindered by surgery.Correct Answer: Observation
2242. (2881) Q5-3382:
![]() |
![]() |
Slide 1 Slide 2
The first radiograph (Slide 1) shows the pelvis of a patient with L3 myelomeningocele at 9 years old. The second radiograph (Slide 2) shows the pelvis of the same patient taken 2 years later. Which of the following factors most likely contributed to the change in the patientâs pelvis:
1) Increased standing activity in therapy
3) Adductor tenotomy
2) Excessive sitting
5) Tethered cord
4) Chiariâs malformation
Note that the radiograph in Slide 1 shows normal hip joints, and the radiograph in Slide 2, which was taken 2 years later, shows symmetric dislocation.
This patient has a tethered cord. The tethered cord caused increased spasticity, resulting in the spontaneous dislocation of the patient's hips during the 2 years between the time the two radiographs were taken.
Correct Answer: Tethered cord
2243. (2882) Q5-3383:
In which of the following molecules is McCune-Albrightâs syndrome due to a mutation:
1) Fibroblast growth factor receptor protein
3) Bone morphogenetic protein
2) GNAS1
5) Collagen type 1
4) Fibrillin
McCune-Albrightâs syndrome (also known as polyostotic fibrous dysplasia) is due to a mutation in GNAS1. GNAS1 is the alpha subunit of GS, which is a protein that links receptors to adenyl cyclase activity.Correct Answer: GNAS1
2244. (2883) Q5-3384:
Which of the following symptoms is not characteristic of McCune-Albrightâs syndrome:
1) Café-au-lait spots with irregular borders
3) Precocious puberty
2) Café-au-lait spots with smooth borders
5) Hyperthyroidism
4) Diabetes mellitus
The café-au-lait spots associated with McCune-Albrightâs syndrome are described as âcoast of Maineâ spots because they have irregular borders. The café-au-lait spots associated with neurofibromatosis are described as âcoast of Californiaâ spots because they have smooth borders.Correct Answer: Café-au-lait spots with smooth borders
2245. (2884) Q5-3385:
A 12-year-old patient with fibrous dysplasia has an increasing limp and progressive bowing in the intertrochanteric and subtrochanteric regions of his femur. Recommended treatment includes:
1) Allograft strut graft in situ
3) Rod fixation in situ
2) Plate fixation in situ
5) Hip arthrodesis in straight position
4) Valgus osteotomy with plate and/or rod
The most important step in treating patients with fibrous dysplasia is to decrease the bending force on the patientâs femur. A valgus osteotomy with a plate or a rod decreases the bending force on the patientâs femur by decreasing the offset from the center of the patientâs body mass. Allografting and plate or rod fixation in situ causes progressive bowing in the patient.Correct Answer: Valgus osteotomy with plate and/or rod
2246. (2885) Q5-3386:
Which of the following bones is the most common site for involvement with fibrous dysplasia:
1) Phalanges of the hand
3) Ulna
2) Radius
5) Femur
-
Humerus
The femur and the tibia are the most common sites of fibrous dysplasia in the appendicular skeleton. Involvement of the humerus is also common. Fibrous dysplasia may also occur in the radius, ulna, and phalanges of the hand; it occurs less frequently in the femur, humerus, and pelvis.Correct Answer: Femur
2247. (2886) Q5-3387:
What is the risk of malignant transformation over the course of a lifetime in a person with fibrous dysplasia:
1) 50%
3) 15%
2) 25%
-
Less than 1%
4) 5%
Fibrous dysplasia is a relatively common disorder. The risk of malignant transformation is less than 1%. Many historic cases of malignant transformation with fibrous dysplasia occurred in patients who also received external beam irradiation.Correct Answer: Less than 1%
2248. (2887) Q5-3388:
The most common malignancy arising from transformation of fibrous dysplasia is:
-
Chondrosarcoma
3) Giant cell tumor
2) Adamantinoma
5) Fibrosarcoma
4) Osteosarcoma
The most common malignancies arising from transformation of fibrous dysplasia are osteosarcoma and malignant fibrous histiocytoma. However, chondrosarcoma, giant cell tumor, adamantinoma, and fibrosarcoma have been reported to result from fibrous dysplasia. Radiation increases the risk of malignant transformation of fibrous dysplasia.Correct Answer: Osteosarcoma
2249. (2888) Q5-3389:
The pattern of genetic transmission of polyostotic fibrous dysplasia is best described as:
1) Autosomal dominant with high penetrance
3) Autosomal recessive
2) Autosomal dominant with low penetrance
5) No genetic transmission (sporadic)
4) Sex-linked dominant
Fibrous dysplasia is a sporadic condition. Although there are a few reports of parents and children having fibrous dysplasia, such reports are rare.Correct Answer: No genetic transmission (sporadic)
2250. (2889) Q5-3390:
![]() |
Slide 1
Which of the following conditions does this computerizd tomography scan (Slide) most likely represent:
1) Brodieâs abscess
3) Fibrous dysplasia
2) Osteosarcoma
5) Enchondromas
4) Osteoid osteoma
This is an osteoid osteoma of the fourth lumbar vertebra in a patient who experienced night pain and relieved the pain with nonsteroidal anti-inflammatory drugs. The location of a sclerotic nidus in the posterior elements of the vertebrae is typical for this disorder. Excision of the osteoid osteoma resulted in prompt pain relief for this patient.Correct Answer: Osteoid osteoma
2251. (2890) Q5-3391:
![]() |
Slide 1
This radiograph (Slide) shows a 9-year-old boy with scoliosis. From which of the following conditions is the boyâs scoliosis likely to have resulted:
1) Marfanâs syndrome
3) Neurofibromatosis
2) Osteogenesis imperfecta
5) Juvenile idiopathic scoliosis
4) Fibrous dysplasia
The patientâs scoliosis is the result of neurofibromatosis. A sharp focal curve over few vertebrae, thinning of apical pedicles, and spindling of the ribs are symptoms of neurofibromatosis that are indicated in the radiograph.Correct Answer: Neurofibromatosis
2252. (3365) Q5-4273:
![]() |
slide 1
A 5-year-old boy presents for examination. He is diagnosed with developmental dysplasia of the hip. Recommended treatment includes:
1) Closed reduction and cast application
3) Femoral osteotomy and closed reduction
2) Traction, closed reduction, and cast
5) Open reduction, femoral osteotomy, and iliac osteotomy
4) Open reduction and iliac osteotomy
At age 5, traction or closed reduction is not likely to produce a stable joint. Femoral shortening is indicated to reduce pressure, reducing the likelihood of avascular necrosis or redislocation. The most likely option to produce a stable joint is open reduction with femoral and iliac osteotomy.Correct Answer: Open reduction, femoral osteotomy, and iliac osteotomy
2253. (3383) Q5-4298:
![]() |
slide 1
The radiograph (Slide) of a 16-month-old toddler is presented. Which diagnosis is most appropriate:
1) Proximal focal femoral deficiency
3) Traumatic hip dislocation
2) Congenital coxa valga
5) Femoral retroversion
4) Developmental dysplasia of the hip
This patient has developmental dysplasia of the hip. The femoral head is delayed in ossifying because of lower contact pressure and the femur is anteverted, not retroverted.Correct Answer: Developmental dysplasia of the hip
2254. (3384) Q5-4299:
![]() |
![]() |
slide 1 slide 2
A 4-year-old girl sustains an injury in a motor vehicle accident. She sustained a femoral artery injury, which was repaired. Her pubic diastasis is 4.5 cm. A radiograph (Slide 1) and clinical photograph (Slide 2) are presented. Which of the following treatment options is recommended:
1) Accept deformity; no weight bearing for 6 to 8 weeks
3) Long leg cast application
2) Spica cast application
5) Open reduction internal fixation
4) Pelvic sling
This is an open-book injury due to direct frontal impact, which presumably also injured the femoral artery directly. A pubic diastasis .2.5 cm should be reduced. Open reduction and internal fixation is the preferred method to accomplish this, using either a wire or a plate. External fixation is also an acceptable option. Casts and sling are more likely to cause pressure sores or to be ineffective.Correct Answer: Open reduction internal fixation
2255. (3385) Q5-4300:
Loeys-Dietz syndrome is caused by a mutation in:
1) Fibrillin -1
3) TGF-beta receptor
2) Fibrillin-2
5) Decorin
4) Collagen type III
Loeys-Dietz syndrome is an autosomal dominant syndrome characterized by arterial tortuosity, aneurysms, hypertelorism, and bifid uvula or cleft palate. Scoliosis, foot deformities, ligamentous laxity, and other findings are often present. The aneurysms have particular risk for rupture at small diameters. This disorder is caused by mutations in genes encoding TGF-beta receptor 1 and 2.Correct Answer: TGF-beta receptor
2256. (3386) Q5-4301:
An 11-year-old female patient with bilateral cavus feet presents with foot pain and callosities on the plantar surface of the foot. She is diagnosed with Charcot-Marie-Tooth disease and may require surgical intervention. During a standing Colemanâs lateral block test, the patientâs hindfoot varus corrects bilaterally when standing on a 1-inch wooden block. Which of the following surgical options is the most appropriate:
1) Plantar fasciotomy
3) Osteotomy of the first metatarsal
2) Plantar-based opening wedge osteotomy of the medial cuneiform
5) Triple arthrodesis
4) Lateral closing wedge osteotomy of the calcaneus
This patient has a typical cavovarus foot. The key in evaluating the treatment options is the standing block test. Because her hindfoot varus is corrected during this test, her hindfoot is flexible and a calcaneal osteotomy is not necessary. Osteotomy of the base of the first metatarsal is also not preferred in such a case because the growth plate becomes at risk for arrest, the procedure requires internal fixation, the second metatarsal head becomes at risk for a stress transfer lesion, and is not located at the side of the deformity. In Charcot-Marie-Tooth disease, the deformity apex generally lies in the tarsometatarsal articulations and the medial cuneiform.Correct Answer: Plantar-based opening wedge osteotomy of the medial cuneiform
2257. (3387) Q5-4302:
Which of the following disorders is not a cause of cavus foot:
1) Charcot-Marie-Tooth disease
3) Myelomeningocele
2) Friedreichâs ataxia
5) Tarsal coalition
4) Poliomyelitis
![]() |
![]() |
All of the disorders mentioned may cause cavus foot except for tarsal coalition.Correct Answer: Tarsal coalition 2258. (3388) Q5-4303:
slide 1 slide 2
Based on the clinical photograph (Slide 1) and radiographs (Slide 2) of this 11-year-old boy, which of the following conditions is demonstrated:
1) Idiopathic scoliosis
3) Tuberous sclerosis
2) Fibrous dysplasia
5) Diastematomyelia
4) Neurofibromatosis 1
This patient has many typical features of neurofibromatosis 1: subcutaneous neurofibromas, large âcoast of Californiaâ cafe-au-lait spot with several others, and a dystrophic curve at a young age. Surgery is recommended.Correct Answer: Neurofibromatosis 1
2259. (3389) Q5-4304:
Which of the following factors predicts an increased risk that a child sustaining a pelvis fracture will incur an unstable fracture:
1) Whether the triradiate cartilages are open or closed
3) Childâs body mass index
2) Childâs body weight
5) Presence of associated injuries
4) Childâs age
The closure of the triradiate cartilage of the acetabulum is associated with a significant increase in the risk of an unstable pelvis fracture, as well as the need for surgical treatment. This seems to signal the change in bone plasticity from pediatric type to adult type.Correct Answer: Whether the triradiate cartilages are open or closed
2260. (3390) Q5-4305:
The mean age of triradiate cartilage closure in girls and boys is:
1) 10 years for girls and 12 years for boys
3) 12 years for girls and 13 years for boys
2) 11 years for girls and 12 years for boys
5) 13.5 years for girls and 14.5 years for boys
4) 12.5 years for girls and 13.5 years for boys
The triradiate cartilage closes at a mean of 12.5 years in girls and 13.5 years in boys. The closure of the triradiate cartilage signals the end of the peak height velocity (growth spurt). This is important for timing of scoliosis treatment and for signaling a change in pelvic fracture patterns from pediatric to adult.Correct Answer: 12.5 years for girls and 13.5 years for boys
2261. (3391) Q5-4306:
Children with unstable pelvis fractures have an increased risk of late pain and dysfunction if which of the following is present:
1) Triradiate cartilages are closed
3) Pelvic asymmetry >1.1 cm after reduction
2) Pelvic asymmetry >0.6 cm after reduction
5) Age is older than 8 years
4) A spica cast is used
Pediatric pelvis fractures are associated with an increased risk of late pain and dysfunction if the asymmetry after reduction is .1.1 cm.Correct Answer: Pelvic asymmetry >1.1 cm after reduction
2262. (3392) Q5-4307:
Which of the following is true regarding ability to remodel after a displaced pediatric pelvic fracture:
1) The pelvis will remodel if the patient is younger than 11 years old.
3) The pelvis will remodel if triradiate cartilages are open.
2) The pelvis will remodel if the patient is younger than 8 years old.
5) No significant remodeling is seen at any age.
4) The pelvis will remodel if the patient is premenarchal.
In pediatric pelvic fractures healing with asymmetry, no significant remodeling is seen at any age.Correct Answer: No significant remodeling is seen at any age.
2263. (3393) Q5-4308:
According to the Delbet classification, a transphyseal fracture of the pediatric proximal femur is considered type:
1) I
3) III
2) II
5) V
4) IV
According to the Delbet classification:
Type I: Transphyseal fracture Type II: Transcervical fracture Type III: Basicervical fracture Type IV: Intertrochanteric fracture
Correct Answer: I
2264. (3394) Q5-4309:
According to the Delbet classification, the risk of avascular necrosis is least with which of the following pediatric hip fractures:
1) Type I
3) Type III
2) Type II
5) Type V
4) Type IV
The risk of avascular necrosis is highest with type I, followed by about equal rates for types II and III (approximately 15 %). The risk of avascular necrosis is low for type IV, intertrochanteric fractures.
Type I Transphyseal Type II Transcervical Type III Basicervical Type IV Intertrochanteric
Correct Answer: Type IV
2265. (3395) Q5-4310:
A 14-year-old boy sustains a hip dislocation in a motor vehicle accident. Recommended treatment is:
1) Closed reduction using skeletal traction
3) Open reduction through a posterior approach
2) Closed reduction using fluoroscopy
5) Percutaneous physeal stabilization followed by closed reduction
4) Open reduction through an open approach
Hip dislocations in young adolescents require a high-energy trauma, which may result in occult injury to the physis. Numerous reports of physeal separation during the reduction are found in the literature. The authors of the largest series recommend gentle closed reduction under fluoroscopy followed by prophylactic stabilization if there is evidence of physeal injury.Correct Answer: Closed reduction using fluoroscopy
2266. (3396) Q5-4311:
Which of the following has not shown a decrease in the success rate of flexible intramedullary nails for femur fractures in children:
1) Age older than 10 years
3) Comminution
2) Weight more than 120 lbs
5) Transverse fracture pattern
4) Long oblique fracture pattern
All of the factors, except for a transverse fracture pattern, have been shown to decrease the chances of success in treating children with femur fractures using flexible intramedullary nails.Correct Answer: Transverse fracture pattern
2267. (3397) Q5-4312:
Which of the following is the best starting point for inserting a rigid femoral intramedullary nail in a 13-year-old boy:
1) Piriformis fossa
3) Apex of the greater trochanter
2) Medial to the tip of the greater trochanter
5) Below the growth plate of the greater trochanter
4) Between the tip and the growth plate of the greater trochanter
Avascular necrosis is a risk if a nail is inserted near the piriformis fossa in a patient younger than 15 years old with open physes. The best way to avoid this risk is to insert the intramedullary nail just lateral to the tip of the greater trochanter.Correct Answer: Between the tip and the growth plate of the greater trochanter
2268. (3398) Q5-4313:
![]() |
slide 1
A patient had an elbow fracture that was openly reduced and internally fixed. The radiographs (Slide) from his first postoperative checkup are presented. Which of the following is a correct assessment:
1) The fixation is adequate.
3) The two pins should have diverged in the proximal fragment.
2) The second pin should have been placed from the medial side.
5) The pins should have been buried under the skin.
4) The pins should have been larger in diameter.
This patient has a lateral condyle fracture. This type of fracture has a tendency to spread unless adequate compression is maintained. Diverging pins in the proximal fragment is the most widely advocated strategy.Correct Answer: The two pins should have diverged in the proximal fragment.
2269. (3399) Q5-4314:
Which of the following femur fractures is best treated with flexible intramedullary nails:
1) A 12-year-old child, 45 kg, transverse fracture of the midshaft
3) An 8-year-old child, 55 kg, transverse fracture of the midshaft
2) An 8-year-old child, 28 kg, midshaft fracture with grade 3 butterfly fragment
5) A 4-year-old child, 20 kg, transverse midshaft fracture with 2 cm shortening
4) An 8-year-old child, 35 kg, oblique fracture 8 cm below the lesser trochanter
Age older than 10 years, weight greater than 50 kg, and length-unstable fractures are associated with poor results after insertion of flexible intramedullary nails. The subtrochanteric region is defined in children as 10% femur length below the lesser trochanter. Children age 5 and younger are best treated with immediate spica cast.Correct Answer: An 8-year-old child, 35 kg, oblique fracture 8 cm below the lesser trochanter
2270. (3400) Q5-4315:
A mutation in the gene for peripheral myelin protein 22 causes which of the following disorders:
1) Friedreich ataxia
3) Cerebral palsy
2) Charcot-Marie-Tooth disease
5) Fragile X syndrome
4) Adrenoleukodystrophy
Type I, or the hypertrophic demyelinating form, is the most common form of Charcot-Marie-Tooth disease. It is due to a mutation in the gene on chromosome 22, which encodes for peripheral myelin protein 22. Type I Charcot-Marie-Tooth disease has a prevalence of approximately 1 in 5,000 individuals.Correct Answer: Charcot-Marie-Tooth disease
2271. (3401) Q5-4316:
Which of the following statements is true of scoliosis in patients with Charcot-Marie-Tooth disease:
1) Scoliosis responds well to bracing.
3) Scoliosis is rarely progressive.
2) Spinal cord monitoring is usually normal.
5) Thoracic kyphosis is less than in idiopathic scoliosis.
4) Thoracic curves more commonly have an apex convex to the left than the right.
Scoliosis is more common in patients with Charcot-Marie-Tooth disease than in the general population (between 10% to 33%). Only 15% of curves are halted by bracing. Scoliosis is commonly associated with thoracic kyphosis and with a left thoracic curve pattern. Intraoperative spinal cord monitoring is often difficult to elicit even at baseline due to the underlying neurologic disorder.Correct Answer: Thoracic curves more commonly have an apex convex to the left than the right.
2272. (3402) Q5-4317:
Risk factors for developmental dysplasia of the hip (DDH) include all of the following except:
1) Oligohydramnios
3) Congenital muscular torticollis
2) Firstborn child
5) Male gender
4) Identical twin with DDH
Developmental dysplasia of the hip is caused by both genetic and mechanical factors. Crowding and laxity are common factors. Females, pregnancies with oligohydramnios, firstborn children, infants with congenital muscular torticollis, and those with an affected identical twin are at increased risk. Also parental hip dysplasia, congenital dislocation of the knee, and breech position increase the risk of DDH.Correct Answer: Male gender
2273. (3403) Q5-4318:
In ultrasound of the infant hip, the alpha angle is defined as:
1) The angle between the acetabular roof and the midline of the pelvis
3) The angle between the center of the femoral head and the lateral wall of the ilium
2) The acute angle between the lateral wall of the ilium and the extension of the acetabular roof
5) The angle between the acetabular roof and the transverse plane
4) The angle of the thigh required to produce subluxation of the hip on the sonogram
The alpha angle is the acute angle between the lateral wall of the ilium and the extension of the acetabular roof. This angle varies inversely with the acetabular index as seen on plain radiographs.Correct Answer: The acute angle between the lateral wall of the ilium and the extension of the acetabular roof
2274. (3404) Q5-4319:
In a sonogram, the normal alpha angle of the neonatal hip measures:
1) <50°
3) <30°
2) <40°
5) >75°
4) >50°
The alpha angle is the acute angle formed by the lateral wall of the ilium and the extension of the acetabular roof. This angle varies inversely with the acetabular index as seen on plain radiographs. An angle >50° is normal.Correct Answer: >50°
2275. (3405) Q5-4320:
Diagnosis of Duchenne muscular dystrophy in a female patient could be explained if the patient had which of the following:
1) Addison disease
3) Trisomy 21
2) Turner syndrome
5) Fragile X syndrome
4) Klinefelter syndrome
Duchenne muscular dystrophy is the most common form of muscular dystrophy, affecting one in 3,300 males. Duchenne muscular dystrophy is an X-linked recessive defect that results from the absence of dystrophin. It may be expressed in females with Turner syndrome, which is the presence of a single X chromosome. Such patients would otherwise be carriers of Duchenne muscular dystrophy but express this recessive disorder because they only have one X chromosome. Duchenne muscular dystrophy would not occur in Klinefelter syndrome, which involves the presence of an extra X chromosome, or in trisomy 21. Steroids can help mitigate the phenotype; therefore Addison disease (hypercortisonism) would not produce the disorder. Fragile X is the most common form of inherited mental retardation but affects only males.Correct Answer: Turner syndrome
2276. (3406) Q5-4321:
Spinal muscular atrophy is due to a mutation in the gene for which of the following proteins:
1) Dystrophin
3) Peripheral myelin protein 22
2) Survival motor neuron
5) Type I collagen
4) Frataxin
Spinal muscular atrophy is due to a mutation in the survival motor neuron gene, which results in loss of many anterior horn cells of the spinal cord. Dystrophin abnormalities cause Duchenne and Becker dystrophies. Peripheral myelin protein 22 is disordered in Charcot-Marie-Tooth disease and frataxin in Friedreich ataxia. Type I collagen disorders cause structural skeletal dysplasias.Correct Answer: Survival motor neuron
2277. (3407) Q5-4322:
Which of the following agents is contraindicated in children with open physes:
1) Pamidronate
3) Etidronate
2) Alendronate
5) Calcium carbonate
4) Teriparatide
Teriparatide, or Forteo (Eli Lilly and Company, Indianapolis, Ind), is contraindicated in children with open physes because of a concern for the risk of osteosarcoma.Correct Answer: Teriparatide
2278. (3408) Q5-4323:
![]() |
slide 1
A 15-month-old toddler, who is neurologically intact, presents with a fracture (Slide). Which of the following is the recommended treatment:
1) Posterior C 1-C 2 fusion
3) Open reduction and screw fixation
2) Posterior occiput-C 2 fusion
5) Reduction and halo vest immobilization
4) C 2 corpectomy and C 1-C 3 fusion anteriorly
This odontoid physeal fracture should be treated by postural reduction and external immobilization. The reduction maneuver is posterior translation with slight axial traction, which may be accomplished by a halter or a halo vest. Immobilization must include a device such as a Minerva cast or a halo vest that can control the head well.Correct Answer: Reduction and halo vest immobilization
2279. (3409) Q5-4324:
![]() |
slide 1
A 17-month-old toddler sustained a femur fracture (Slide) in a fall from a height. Which of the following is the best treatment method:
1) Pavlik harness
3) Femoral skeletal traction
2) Spica cast
5) Flexible intramedullary nails
4) External fixation
This toddlerâs fracture shows minimal shortening. Spica cast treatment is ideal for fractures of the femur in children younger than school age because of their portability, overgrowth, remodeling, and lack of implant to remove. A Pavlik harness does not control a child beyond the age of a few months. Femoral skeletal traction, external fixation, and flexible intramedullary nails are more invasive than is warranted.Correct Answer: Spica cast
2280. (3410) Q5-4325:
![]() |
slide 1
The lesion shown in the radiograph (Slide 1) most likely represents which of the following processes:
1) Aneurysmal bone cyst
3) Unicameral bone cyst
2) Fibrous dysplasia
5) Fibrous cortical defect
4) Osteomyelitis
The lesion shown in the radiograph is a unicameral bone cyst. The diagnosis was confirmed by aspiration and the subsequent response, filling in after autogenous marrow (Slide 2).
An aneurysmal bone cyst is typically more septated and expansile in width, fibrous dysplasia has a more blurred zone of transition and ground-glass appearance, and a fibrous cortical defect is more eccentrically placed.
![]() |
![]() |
![]() |
Correct Answer: Unicameral bone cyst 2281. (3411) Q5-4326:
slide 1 slide 2
A clinical photograph (Slide 1) and radiographs (Slide 2) of a 13-year-old girl, who is neurologically normal, are presented. She does not report any pain. The most likely diagnosis is:
1) Fibrous dysplasia
3) Idiopathic scoliosis
2) Blue-rubber bleb nevus syndrome
5) Klippel-Trenaunay syndrome
4) Neurofibromatosis-1
This patient has neurofibromatosis-1. She has a dystrophic scoliosis that is sharply angulated and involves only four vertebrae (Cobb levels T12-L3). The pedicles are thinned, and the endplates are scalloped. She also has subcutaneous neurofibromas.Correct Answer: Neurofibromatosis-1
2282. (3412) Q5-4327:
![]() |
slide 1
An 11-year-old girl presents with a limp. She has no history of trauma, infection, or neurologic disorder. She does not report any pain. Recommended treatment includes:
1) Hip arthrodesis
3) Cemented total hip arthroplasty
2) Uncemented total hip arthroplasty
5) No treatment
4) Resurfacing hip arthroplasty
This patient has untreated developmental dysplasia of the hip. Her limp cannot be eliminated. If she is not experiencing pain, then no treatment is indicated.Correct Answer: No treatment
2283. (3413) Q5-4328:
![]() |
slide 1
The foot pictured in this clinical photograph (Slide) represents:
1) Calcaneovalgus foot
3) Calcaneus foot
2) Clubfoot
5) Skewfoot
4) Congenital vertical talus
The foot presented in the clinical photograph represents congenital vertical talus. Note the equinus of the hindfoot, calcaneus of the forefoot, and the crease in the sinus tarsi. In patients with a calcaneovalgus foot, the hindfoot is in calcaneus, not equinus.Correct Answer: Congenital vertical talus
2284. (3414) Q5-4329:
Scapulothoracic fusion is most commonly indicated to help improve function in which of the following conditions:
1) Duchenne muscular dystrophy
3) Facioscapulohumeral dystrophy
2) Beckerâs muscular dystrophy
5) Congenital muscular dystrophy
4) Charcot-Marie-Tooth disorder
Facioscapulohumeral dystrophy is a rare disorder inherited in an autosomal dominant fashion. Thirty percent of affected individuals have a new mutation. The genetic abnormality is found on chromosome 4, with a decreased number of D4Z4 tandem repeats, but this does not appear to code for a protein product. In this condition, selective weakness of the serratus anterior, trapezius, and rhomboid muscles is present. Therefore, the scapula is not effectively stabilized against the trunk during use.
Although the deltoid is relatively spared, it cannot work well due to a hypermobile scapula. Fusion of the scapula to the thorax improves range of abduction in this condition.Correct Answer: Facioscapulohumeral dystrophy
2285. (3415) Q5-4330:
![]() |
slide 1
A 15-year-old girl is experiencing progressive weakness. She is unable to abduct her arms above 100° (Slide). She has a progressive lordosis. Her facial expression is flat, and she cannot smile. Her mother has the same constellation of findings. No scoliosis is present. She and her mother are able to walk independently. The most likely diagnosis is:
1) Parsonage-Turner syndrome
3) Facioscapulohumeral dystrophy
2) Friedreich ataxia
5) Nemaline rod myopathy
4) Bellâs palsy
This patient most likely has facioscapulohumeral dystrophy. With a frequency of 1:20,000, it is a rare disorder inherited in an autosomal-dominant fashion. The genetic abnormality is found on chromosome 4, with a decreased number of D4Z4 tandem repeats, but this does not appear to code for a protein product. Scoliosis is not present. In this condition, selective weakness of the serratus anterior, trapezius, and rhomboid muscles is present. Therefore, the scapula is not effectively stabilized against the trunk during use.Correct Answer: Facioscapulohumeral dystrophy
2286. (3416) Q5-4331:
![]() |
slide 1
A 10-year-old child, who has no history of fever, trauma, or infection, presents with minimal pain and a Trendelenburg gait on the left (Slide). Which of the following is the most likely diagnosis:
1) Aneurysmal bone cyst
3) Fibrous dysplasia
2) Chronic osteomyelitis
5) Enchondroma
4) Unicameral bone cyst
This patient has fibrous dysplasia. The diffuse nature of the changes over a long portion of the bone, which includes blurring and thinning of the cortex, are classic signs of the disorder.Correct Answer: Fibrous dysplasia
2287. (3417) Q5-4333:
A patient who does not walk outdoors or independently but can walk with a walker in physical therapy is listed as what level according to the Gross Motor Function Measure (GMFM):
1) Level 1
3) Level 3
2) Level 2
5) Level 5
4) Level 4
The GMF Classification System (GMFCS) is a concise way of expressing overall motor ability. According to this system, level 1 is walking and running indoors and out with impaired speed, level 2 is impairment in running on uneven surfaces, level 3 is ambulation indoors and out with assistive devices, and level 4 is limited walking with a walker, and level 5 is nonambulatory.Correct Answer: Level 4
2288. (3418) Q5-4334:
![]() |
![]() |
slide 1 slide 2
The patient shown in the clinical photograph (Slide 1) and radiograph (Slide 2) has which of the following conditions:
1) Klippel-Trenaunay-Weber syndrome
3) Proteus syndrome
2) Neurofibromatosis-1
5) Maffucci syndrome
4) Gorham disease
This patient has Klippel-Trenaunay-Weber syndrome. This syndrome is characterized by a triad of cutaneous nevi, varicose veins, and limb overgrowth in length and/or width.Correct Answer: Klippel-Trenaunay-Weber syndrome
2289. (3419) Q5-4335:
![]() |
![]() |
slide 1 slide 2
An 8-year-old girl with no history of fever, illness, or weakness presents with an increasing spinal deformity. She was born in Asia. The patient is neurologically normal. Based on radiographs (Slide 1) and magnetic resonance images (Slide 2), the most likely diagnosis is:
1) Pyogenic spondylitis
3) Compression fractures
2) Tuberculous spondylitis
5) Scheuermann kyphosis
4) Congenital kyphosis
This patient has congenital kyphosis. The relative disk destruction with vertebral preservation argues against tuberculosis. The anterior bony wedging is atypical for remote bacterial infection. Anterior fusion is not seen in patients with Scheuermann kyphosis (rarely after maturity) or in patients with compression fractures.Correct Answer: Congenital kyphosis
2290. (3420) Q5-4336:
![]() |
slide 1
An 8-year-old boy presents with progressive deformity. Recommended management includes:
1) Observation
3) Posterior fusion in situ
2) Bracing
5) Anterior and posterior fusion
4) Anterior fusion in situ
This patient has a type II kyphosis (anterior bar). Because he is only 8 years old and the curve has progressed, further increase is likely. Observation is not indicated. Bracing has no influence on congenital curves. The kyphosis is >55° (78°) so a posterior fusion may not effectively halt the growth of these vertebral bodies in a posterior direction. Anterior and posterior fusion is the most likely procedure to control the curve.Correct Answer: Anterior and posterior fusion
2291. (3421) Q5-4337:
A child with congenital below-elbow amputation is best fit with an active prosthesis at which age:
1) 3 months
3) 9 months
2) 6 months
5) 24 months
4) 12 months
A patient with congenital below-elbow amputation has sensation/proprioception in the limb and is able to perform most activities without assistance. Prostheses may interfere with crawling. Passive devices should not be started until walking begins, and active devices started at 2 to 4 years.Correct Answer: 24 months
2292. (3422) Q5-4338:
Which of the following is the most specific feature of congenital vertical talus that distinguishes it from other deformities:
1) Hindfoot equinus
3) Talonavicular dislocation
2) Forefoot adduction
5) Increased talonavicular angle
4) Peroneal tightness
The essential lesion in congenital vertical talus is a dorsolateral dislocation of the talonavicular joint.Correct Answer: Talonavicular dislocation
2293. (3423) Q5-4339:
A 14-year-old boy sustains a hip dislocation in a motor vehicle accident. With closed reduction, he is at risk for all of the following except:
1) Physeal separation
3) Degenerative joint disease
2) Entrapped osteochondral fragment
5) Protrusio acetabulae
4) Avascular necrosis
With a hip dislocation in a young adolescent, occult injury to the physis may occur. Reports of physeal separation during the reduction, as well as entrapped osteochondral fragments appear in the literature. Degenerative joint disease may eventually develop due to the cartilage injury. Avascular necrosis risk is approximately 10%.Correct Answer: Protrusio acetabulae
2294. (3424) Q5-4340:
A 12-year-old boy sustains a distal diaphyseal femur fracture with a large butterfly fragment. He weighs 110 lb. Which of the following treatments is most appropriate and best able to preserve alignment:
1) Flexible intramedullary nails
3) Submuscular plate fixation
2) Immediate hip spica cast
5) Traction for 1 week followed by functional cast-brace
4) Retrograde nail
A large butterfly fragment as well as the age and weight of the patient have been demonstrated to decrease the chances of success in treating children with femur fractures using flexible intramedullary nails. For similar reasons, the fracture is not suitable for an immediate spica or traction followed by functional bracing. The retrograde nail is not indicated due to open physis.
Submuscular plate fixation is the best option.Correct Answer: Submuscular plate fixation
2295. (3425) Q5-4341:
Which of the following risks is associated with inserting a rigid femoral intramedullary nail through the piriformis fossa in a 10- to 14-year-old child:
1) Obligatory varus deformity at the fracture
3) Avascular necrosis
2) Trochanteric overgrowth
5) Recurvatum
4) Chondrolysis
Avascular necrosis is a risk if a nail is inserted through the piriformis fossa in a patient younger than 15 years of age with open physes.Correct Answer: Avascular necrosis
2296. (3426) Q5-4342:
Which of the following is a description of a closed kinetic chain exercise:
1) The distal ends of a limb are against fixed resistance
3) The distal limb exerts force against an elastic chain
2) The distal ends of a limb are free to travel in space
5) Two limbs move together
4) The limb is suspended from a chain
A closed kinetic chain exercise is one in which the ends of a limb are fixed from free movement by resistance.Correct Answer: The distal ends of a limb are against fixed resistance
2297. (3427) Q5-4343:
The action of bisphosphonates is primarily upon which component of bone:
1) Osteoclast
3) Cartilage
2) Osteoblasts
5) Nutrient vessel
4) Periosteum
Bisphosphonates act to inhibit osteoclasts, thereby decreasing resorption and increasing bone mineral density in many conditions.Correct Answer: Osteoclast
2298. (3428) Q5-4344:
Which of the following is not a common finding in patients with Prader-Willi syndrome:
1) Short stature
3) Scoliosis
2) Developmental delay
5) Arachnodactyly
4) Osteopenia
Prader-Willi syndrome is characterized by early hypotonia, short stature, severe obesity, developmental delay, scoliosis, and osteopenia.Correct Answer: Arachnodactyly
2299. (3429) Q5-4345:
![]() |
slide 1
Which of the following disease processes is demonstrated in the radiograph (Slide):
1) Pigmented villonodular synovitis
3) Developmental dysplasia of the hip
2) Slipped capital femoral epiphysis
5) Multiple epiphyseal dysplasia
4) Legg-Calve-Perthes disease
The left femoral epiphysis shows avascular changes characteristic of Legg-Calve-Perthes disease without slippage. Although developmental dysplasia of the hip may also be complicated by avascular changes, the femoral neck and acetabulum would manifest more longstanding growth alterations as well. The contralateral hip would be symmetric in patients with multiple epiphyseal dysplasia.Correct Answer: Legg-Calve-Perthes disease
2300. (3430) Q5-4346:
![]() |
slide 1
Which of the following accurately describes this radiograph (Slide):
1) Catterall 2
3) Lateral pillar C
2) Catterall 3
5) Salter-Thompson A
4) Stulberg 3
According to Herringâs classification, this radiograph demonstrates a lateral pillar C because the lateral pillar is collapsed by more than 50%.Correct Answer: Lateral pillar C
2301. (3431) Q5-4347:
![]() |
slide 1
An 11-year-old girl presents with bilateral painful flatfeet. A computed tomography image is shown (Slide). The most likely diagnosis is:
1) Idiopathic planovalgus feet
3) Degenerative joint disease
2) Talocalcaneal coalition
5) Osteochondritis dissecans
4) Osteoid osteoma
This patient has evolving medial talocalcaneal facet coalition. The computed tomography is remarkable for obliquity of the medial facet, along with irregularity and narrowing of this facet, which is not evident on plain radiographs. Normally the medial subtalar facet should be parallel to the posterior facet. Treatment options for this patient include immobilization, resection, or subtalar fusion.Correct Answer: Talocalcaneal coalition
2302. (3455) Q5-4383:
Which of the following is not a common feature of constriction band syndrome:
1) Annular rings
3) Digital amputation
2) Fenestrated syndactyly
5) Hip dislocation
4) Equinovarus foot
Constriction band syndrome is not commonly associated with developmental dislocation of the hip. However, annular rings, fenestrated syndactyly, digital amputation, and equinovarus foot are often seen in patients with constriction band syndrome.Correct Answer: Hip dislocation
2303. (3474) Q5-4412:
An 8-year-old child presents with a mass on the posteromedial side of the popliteal fossa. The mass, which has been present for more than 1 month, is nontender and moderately soft. The knee examination is stable. No knee effusion is present, and the patient has no signs or symptoms of infection. Radiographs are normal. Which of the following is the next step to aid in diagnosis:
1) Magnetic resonance imaging
3) Computed tomography
2) Bone scan
5) Biopsy
4) Transillumination
This patient demonstrates the classic presentation of a popliteal cyst. Transillumination often confirms the diagnosis of popliteal cyst, thereby avoiding the use of advanced imaging.Correct Answer: Transillumination
2304. (3492) Q5-4434:
Which of the following features is not associated with congenital muscular torticollis:
1) Developmental dysplasia of the hip
3) Flattening of the occiput contralateral to the tight sternomastoid
2) Metatarsus adductus
5) Imperforate anus
4) Asymmetrical facial features
Congenital muscular torticollis is caused by in-utero molding, as are developmental dysplasia of the hip and metatarsus adductus. Contralateral occipital flattening and asymmetrical facial features develop secondarily with torticollis. Imperforate anus is associated with congenital hemivertebrae but not with muscular torticollis.Correct Answer: Imperforate anus
2305. (3495) Q5-4437:
A 3-month-old female infant presents with the dorsum of the foot almost in contact with the anterior tibia. She has no pain with movement; plantar flexion and inversion of the feet are not restricted, but there is difficulty in manipulating the foot to the neutral position. She is normal otherwise. Which of the following treatments is most appropriate:
1) Observation ± passive stretching
3) Botulinum toxin to peroneals
2) Serial casting
5) Surgical reduction
4) Serial casting and then surgery
This is a patient with positional calcaneovalgus, which is a more common condition than congenital vertical talus (CVT). Patients with such deformity should be evaluated carefully and differentiated from the more serious CVT. In calcaneovalgus foot, the problem is in the ankle, which is in calcaneus position. The arch itself may be normal. In CVT, the ankle is normal and the problem is in the foot. The foot is essentially plantigrade, but the arch is reversed. Typically, a deep crease in the sinus tarsi and some forefoot abduction are present.
No difference was found between the calcaneovalgus feet that underwent manipulation and casting versus observation alone, when assessed at 3 to 11 years of follow-up. Surgery is never required for a positional calcaneovalgus deformity, which is not true for CVT. Long-term prognosis of positional calcaneovalgus is excellent.
Correct Answer: Observation ± passive stretching
2306. (3496) Q5-4439:
The best way to diagnose dysplasia epiphysealis hemimelica, in addition to history and physical, is:
1) Bone scan
3) Biopsy
2) Ultrasound
5) Bacterial culture
4) Plain radiograph
Dysplasia epiphysealis hemimelica, or Trevor disease, is an epiphyseal osteochondroma. The characteristic location, continuity with the epiphysis and alteration in growth of the epiphysis, is the most specific confirmation of the disease. Histology is less specific, as are bone scan and ultrasound.Correct Answer: Plain radiograph
2307. (3498) Q5-4443:
Which of the following commonly used statements regarding the pelvis in patients with bladder exstrophy is true:
1) The forces across the hip joints are increased in patients with bladder exstrophy.
3) Significant limp persists throughout life in patients with bladder exstrophy.
2) Athletic ability is significantly compromised in patients with bladder exstrophy.
5) Urinary continence is not possible in patients with bladder exstrophy.
4) The pelvis is more vulnerable to fracture in patients with bladder exstrophy.
In patients with bladder exstrophy, computerized modeling shows that the forces across the hip are increased by approximately 30%. However, limp is not significant and the pelvis is not more prone to fracture. Most patients can be made continent through surgery.Correct Answer: The forces across the hip joints are increased in patients with bladder exstrophy.
2308. (3512) Q5-4465:
A 1-year-old child requires evaluation of limb length inequality. The surgeon notices a disproportional increase in length, as well as width, of the ipsilateral upper and lower extremity. No other physical abnormalities are evident. Recommended treatment includes which of the following:
1) Ultrasound periodically throughout childhood
3) MRI of the affected extremities
2) Brain magnetic resonance imaging (MRI)
5) Angiogram of the affected limbs
4) Spinal ultrasound
Idiopathic hemihypertrophy is associated with an increased incidence of intra-abdominal tumors such as Wilmsâ tumor and hepatoblastoma. Periodic screening by ultrasound throughout childhood is recommended.Correct Answer: Ultrasound periodically throughout childhood
2309. (3571) Q5-4567:
Which of the following statements describes the growth plate biomechanics of the distal femur:
1) The collateral ligaments protect the distal femur.
3) The anterior cruciate ligaments protect the distal femur.
2) The posterior cruciate ligament protects the distal femur.
5) The distal femur is not protected by any ligaments.
4) The patellar ligament protects the distal femur.
Whereas the the proximal tibial physis is protected by the collateral ligaments and tibial tubercle epiphysis, the distal femoral physis is vulnerable to injury because it is not protected by any ligaments.Correct Answer: The distal femur is not protected by any ligaments.
2310. (3572) Q5-4568:
![]() |
![]() |
slide 1 slide 2
A 200-lb 13-year-old boy sustained a fracture while playing football. His radiographs are presented (Slide 1). Which of the following treatments should be attempted first:
1) Closed reduction and internal fixation
3) Open reduction and plate fixation
2) Open reduction and screw fixation
5) Arthroscopically assisted reduction and internal fixation
4) External fixation
The radiographs show a Salter II fracture with a large metaphyseal fragment. A high likelihood exists for successful closed reduction, and the metaphyseal fragment should allow rigid fixation. This patient received closed reduction and was internally fixed with three percutaneous 7.3-mm screws (Slide 2). He did not require a cast.Correct Answer: Closed reduction and internal fixation
2311. (3573) Q5-4569:
![]() |
An 8-year-old boy was injured by a lawnmower. The resulting proximal tibial injury presented in the radiograph (Slide) is classified as a:
1) Salter I injury
3) Salter III injury
2) Salter II injury
5) Salter V injury
4) Salter IV injury
A Salter IV injury to the proximal tibia is apparent in the patientâs radiograph. The fracture traverses the epiphysis, physis, and metaphysis medial to the tibial spine. No injury to the lateral portion of the plateau is present. Incidentally, the distal femoral injury is also classified as Salter IV.Correct Answer: Salter IV injury
2312. (3574) Q5-4570:
![]() |
A 12-year-old boy twisted his knee while riding a bicycle. Based on his radiographs (Slide), which of the following is the most appropriate diagnosis:
1) Patellar dislocation
3) Tibial spine avulsion
2) Traumatic osteochondritis dissecans
5) Tibial tubercle fracture
4) Anterior cruciate ligament tear
This patient has a tibial spine avulsion, classified as type III by McKeever and Meyers. Treatment options include closed immobilization or arthroscopic or open reduction and internal fixation.Correct Answer: Tibial spine avulsion
2313. (3575) Q5-4571:
What is the most common cause of intoeing in children with bilateral cerebral palsy:
1) Internal pelvic rotation
3) Increased tibial torsion
2) Internal hip rotation
5) Forefoot adduction
4) Hindfoot rotation
The most common cause of intoeing in children with bilateral spasticity is internal hip rotation. For children with hemiplegia, the most common cause of intoeing is tibial torsion. In some patients, several causes may coexist to cause the condition.Correct Answer: Internal hip rotation
2314. (3576) Q5-4572:
Risk factors for superior mesenteric artery syndrome after adolescent idiopathic scoliosis surgery include all of the following except:
1) Decreased correction of thoracic curve with preoperative bending
3) Lenke lumbar modifier B or C
2) Body mass index (BMI) below the 25th percentile
5) Use of iliac crest bone graft
4) Staged surgical correction
Superior mesenteric artery syndrome occurs more often in patients with decreased BMI, larger and stiffer thoracic curves, lumbar apical translation of Lenke B or C, and two staged procedures.Correct Answer: Use of iliac crest bone graft
2315. (3577) Q5-4573:
![]() |
An 8-year-old girl fractures her right femur when starting a sprint. Based on her radiograph (Slide), which of the following is the most likely diagnosis:
1) Unicameral bone cyst
3) Osteogenic sarcoma
2) Aneurysmal bone cyst
5) Normal pre-existing bone
4) Fibrous dysplasia
This patient has signs of a pre-existing fibrous dysplasia lesion, including a central expansion of the subtrochanteric region with a âground-glassâ (not lucent) appearance. She was diagnosed with fibrous dysplasia.Correct Answer: Fibrous dysplasia
2316. (3578) Q5-4574:
![]() |
An 8-year old girl with fibrous dysplasia fractures her femur (Slide). Which of the following procedures is the best treatment option:
1) Femoral traction followed by spica cast
3) Allograft strut and spica cast
2) Curettage, bone graft, and spica cast
5) Hip screw with long side plate
4) External fixation
This lesion requires mechanical support because it is vulnerable to malunion and further deformity. Bone graft would be rapidly reabsorbed and has no lasting value in this patient. A screw with a long side plate that extends well into normal bone is the best treatment option.Correct Answer: Hip screw with long side plate
2317. (3579) Q5-4575:
![]() |
The patient presented in the radiograph (Slide) has a slight, painless limp. Which of the following is the most likely diagnosis:
1) Ollier disease
3) Fibrous dysplasia
2) Osteogenesis imperfecta
5) Fibrous cortical defects
4) Neurofibromatosis
This patient has multiple âground-glassâ lesions and one cystic lesion, as well as bowing in the subtrochanteric region. The patient was diagnosed with fibrous dysplasia.Correct Answer: Fibrous dysplasia
2318. (3580) Q5-4576:
A 14-year-old boy sustains an avulsion of the tibial tubercle while playing basketball. The fracture extends vertically into the joint, crossing the proximal tibial articular surface. He is at greatest risk for which of the following complications:
1) Peroneal nerve injury
3) Popliteal artery disruption
2) Tibial nerve injury
5) Femoral nerve palsy
4) Anterior leg compartment syndrome
Fracture of the tibial tubercle has been reported to lead to compartment syndrome, presumably due to bleeding from the geniculate vessels and the exposed bone. This would not be expected from such a proximal fracture, but orthopedic surgeons should be aware of the possible association.Correct Answer: Anterior leg compartment syndrome
2319. (3581) Q5-4577:
An infant presents with idiopathic scoliosis. Which of the following factors indicates observation as the treatment of choice:
1) Age younger than 1 year
3) Rib-vertebra phase 2
2) Rib-vertebral angle difference >20°
5) Left thoracic curve
4) Curve of 47°
Rib-vertebral angle difference >20° rib-vertebra phase 2, and curves >45° indicate a likely progression. The majority of infantile curves are left thoracic, so this is not a factor. A patient younger than 1 year of age has a higher chance of spontaneous resolution than an older patient. Therefore, absent other risk factors, observation is the initial recommend treatment.Correct Answer: Age younger than 1 year
2320. (3582) Q5-4578:
Which of the following factors predicts a lesser degree of impairment of pulmonary function in patients with adolescent idiopathic scoliosis:
1) Increased Cobb angle
3) Increased cephalad apex of the curve
2) Decreased number of vertebrae in the curve
5) Pectus excavatum in addition to the scoliosis
4) Decreased thoracic kyphosis
An increased Cobb angle, increased cephalad apex of the curve, decreased thoracic kyphosis, and pectus excavatum in addition to the scoliosis are factors that predict a greater degree of pulmonary impairment. A lower number of vertebrae in the curve signals less impairment.Correct Answer: Decreased number of vertebrae in the curve
2321. (3583) Q5-4579:
All except which of the following neurologic/muscular disorders can present undiagnosed in a patient with scoliosis:
1) Friedreichâs ataxia
3) Syringomyelia
2) Charcot-Marie-Tooth disease
5) Spinal cord tumor
4) Duchenne muscular dystrophy
Scoliosis can be caused by any neurologic disorder that affects trunk balance. Scoliosis is seen in most patients with Friedreichâs ataxia, a disorder in which patients present with an ataxic gait in preadolescence or adolescence. The curve becomes significant at about the same time as the ataxia. Scoliosis can also be seen in 10% of patients with Charcot-Marie-Tooth disease. This condition is characterized by cavus feet, intrinsic atrophy, and occasional hip dysplasia; no significant pain or clumsiness is present.
Scoliosis is seen in many patients with syringomyelia. The syrinx and scoliosis both develop silently with no noticeable weakness until both the syrinx and the curve are significant. Spinal cord tumor may present with scoliosis in an ambulatory patient.
Duchenne muscular dystrophy leads to scoliosis in most patients, but the patients become nonambulatory several years before the curve develops.Correct Answer: Duchenne muscular dystrophy
2322. (3584) Q5-4580:
All except which of the following structural disorders often causes scoliosis and presents undiagnosed in patients:
1) Marfan syndrome
3) Osteogenesis imperfecta
2) Ehlers-Danlos syndrome
5) Achondroplasia
4) Loeys-Dietz syndrome
Although achondroplasia causes kyphosis, it is not associated with scoliosis to a significant degree. Marfan syndrome, Ehlers-Danlos syndrome, osteogenesis imperfecta, and Loeys-Dietz syndrome (a defect in TGF-beta receptor protein) are frequently associated with scoliosis.Correct Answer: Achondroplasia
2323. (3585) Q5-4581:
Which of the following levels of evidence should be assigned to a prospective, randomized therapeutic study with 80% follow-up:
1) Level I
3) Level III
2) Level II
5) Level V
4) Level IV
A study may be level I as long as it has at least 80% follow-up.Correct Answer: Level I
2324. (3586) Q5-4582:
Which of the following levels of evidence should be assigned to a meta-analysis that covers all level I primary studies:
1) Level I
3) Level III
2) Level II
5) Level V
4) Level IV
A meta-analysis is considered level I evidence as long as all of the primary studies included in the meta-analysis are level
I.Correct Answer: Level I
2325. (3587) Q5-4583:
A prospective comparative study should be assigned which level of evidence:
1) Level I
3) Level III
2) Level II
5) Level V
4) Level IV
A comparative study is one in which patients treated one way are compared with patients treated in another manner at the same institution. As long as the study is prospective, it can be assigned level II.Correct Answer: Level II
2326. (3588) Q5-4584:
A case-control study should be assigned which level of evidence:
1) Level I
3) Level III
2) Level II
5) Level V
4) Level IV
A case-control study is considered level III evidence, as is a retrospective comparison study or a meta-analysis in which the lowest level of primary study is level III.Correct Answer: Level III
2327. (3589) Q5-4585:
![]() |
The radiographs (Slide) of a 13-year-old-boy are presented. The radiographs demonstrate characteristics of which of the following conditions:
1) Osteoid osteoma
3) Congenital dysplasia
2) Tibial hemimelia
5) Osteomyelitis
4) Fracture growth arrest
The radiographs show characteristics of a growth arrest from a medial distal tibial physeal fracture, either a Salter III or Salter IV. The fracture was not internally fixed, and the residual vertical fracture line and step-off are still visible. A growth plate bar developed and Park-Harris growth lines are seen on the lateral side of the bar. Some of the physis is open medially and, especially, laterally. The fibula has continued to grow.Correct Answer: Fracture growth arrest
2328. (3590) Q5-4586:
![]() |
![]() |
An 11½ -year-old boy sustained a Salter IV fracture of the distal tibial physis. The fracture was treated in a cast. Two years later, his radiographs (Slide) are presented. The patient is starting to notice a deformity. Which of the following treatment plans is recommended:
1) Medial physeal bar resection
3) Right distal tibial and fibular epiphyseodesis
2) Right distal fibular epiphyseodesis
5) Right distal tibial and fibular osteotomy, and bilateral distal tibial and fibular epiphyseodesis
4) Left distal tibial and fibular epiphyseodesis
This patient has an unacceptable deformity, 2 cm of current shortening, and significant growth remaining. Only right distal tibial and fibular osteotomy, and bilateral distal tibial and fibular epiphyseodesis addresses all of the complications. Bar resection is not uniformly successful and would not correct existing deformity and length inequality, since there is minimal physis medial to the bar that could produce correction.Correct Answer: Right distal tibial and fibular osteotomy, and bilateral distal tibial and fibular epiphyseodesis
2329. (3591) Q5-4587:
![]() |
The radiographs (Slide) indicate which of the following diagnoses:
1) Congenital dysplasia of the distal humerus
3) Supracondylar humeral fracture
2) Osteogenesis imperfecta
5) Normal elbow
4) Lateral condylar humeral fracture
The radiographs indicate a supracondylar fracture of the distal humerus. The anterior humeral line should bisect the capitellum on the lateral view. The anterior and posterior fat pads are elevated. The fracture line may be seen on the anteroposterior and the lateral radiographs, although it is subtle. The cortical break of the coronoid fossa is evident. Surgeons must recognize these fractures so that the patient and family may be given proper activity restrictions and expectations for improvement.Correct Answer: Supracondylar humeral fracture
2330. (3592) Q5-4588:
![]() |
A 5-year-old boy was imaged after being struck by a car. His radiographs (Slide) indicate which of the following diagnoses:
1) Buckle fracture of the proximal humeral metaphysis
3) Salter II fracture of the proximal humeral physis
2) Salter I fracture of the proximal humeral physis
5) Normal humerus
4) Salter IV fracture of the proximal humeral physis
The proximal humeral physis has two planes and two ossification centers (in this way, it is analogous to the proximal femur). When seen in some projections, the more distal extent of the physis may produce a lucent line. The smooth nature and characteristic location of the line should allow surgeons to make the correct interpretation. The two ossification centers of the humeral head may be misinterpreted as a fracture.
This patient had a greenstick fracture of the clavicle (not included on this field), which was not noticed, and a humeral fracture was incorrectly diagnosised.
Correct Answer: Normal humerus
2331. (3593) Q5-4589:
The largest available free muscle flap comes from which of the following muscles:
1) Gracilis muscle
3) Latissimus dorsi muscle
2) Serratus anterior muscle
5) Brachialis muscle
4) Rectus abdominis muscle
The latissimus dorsi provides the largest available free muscle flap. The gracilis provides a long muscle slip and can be innervated and used for certain innervated muscle transfers. The serratus anterior muscle is thin and can be used when bulk is undesirable.Correct Answer: Latissimus dorsi muscle
2332. (3594) Q5-4590:
![]() |
![]() |
![]() |
slide 1 slide 2 slide 3
A 12-year-old boy presents for evaluation of a spinal curvature (Slide 1). Which of the following is the most likely cause of the spinal curve:
1) Idiopathic scoliosis
3) Osteoid osteoma
2) Neurofibromatosis
5) Cerebral palsy
4) Limb length inequality
This patient has a 2.5 cm shortening of the left lower extremity, which can be seen from the uneven pelvic height on the radiograph. The shortening produces a compensatory curve. No rotation is present within the curve. Scanogam (Slide 2) documents the inequality.
When the patient stands on a 2.5-cm lift, the curve is eliminated (Slide 3). Correct Answer: Limb length inequality
2333. (3595) Q5-4591:
![]() |
In the embryo, cells from the ectodermal layer give rise to which elements of the spine:
1) Spinal cord
3) Paraspinous muscle
2) Vertebral body
5) Annulus fibrosis
4) Nucleus pulposus
The spine is formed from all three elements of the embryonic plate (Slide). The ectoderm develops into a groove, folding into a tube that becomes the spinal cord. The endoderm gives rise to the notochord, which serves as the organizing structure for the spinal column. The notochord persists as the nucleus pulposus. The mesoderm gives rise to muscles and bony elements of the spinal column.Correct Answer: Spinal cord
2334. (3596) Q5-4592:
How many somites are involved in formation of each vertebral body of the thoracolumbar spine:
1) None
3) Two
2) One
5) Four
4) Three
The somites are repeating groups of axial mesodermal cells that give rise to the osseous and musculoligamentous elements of the spine during embryonic formation. Each somite divides and contributes to two adjacent vertebral bodies.Correct Answer: Two
2335. (3597) Q5-6312:
A 6-year-old boy sustains a supracondylar fracture of the humerus. The 2 fragments are not completely displaced, but there is some overlap of the medial column and a gap on the lateral column of the distal humerus. Baumannâs angle measures 85°. The alignment on the lateral film shows no significant translation, but approximately 15° of increased extension. The recommended treatment is:
1) Accept this and treat in a long arm cast
3) Closed reduction with the elbow in extension to better monitor the angulation
2) Closed reduction with supination of the forearm and application of long arm cast
5) Open reduction and medial and lateral plate fixation
4) Closed reduction and percutaneous pin fixation
Closed reduction should allow regain of alignment and percutaneous pin fixation will allow it to be maintained.
The elbow is in 10°-15° of varus and this will be an objectionable appearance in the future. Supination will increase the varus.
Extension will exacerbate the deformity seen on the lateral and will cause further loss of contact of the fracture fragments. Medial and lateral plate fixation is needed in adolescents and adults with intercondylar fractures to allow early range of movement but is excessive treatment for this fracture in young children.
![]() |
![]() |
Correct Answer: Closed reduction and percutaneous pin fixation 2336. (3603) Q5-6485:
slide 1 slide 2
The 7-year-old patient shown in this image (Slide 1) is most likely presenting with:
1) Atlas fracture
3) Facet subluxation
2) Spondylolysis of the axis
5) Fracture and subluxation
4) Fracture of C 3
This patient has a spondylolisthesis of the axis, which is also known as a Hangmanâs fracture. Additionally, a subluxation of the C 2-C 3 facet is present on the right side. Such a subluxation often accompanies a Hangmanâs fracture. The comparison image (Slide 2) of the opposite facet is shown as a reference to demonstrate the normal alignment.Correct Answer: Fracture and subluxation
2337. (3604) Q5-6486:
![]() |
![]() |
slide 1 slide 2
Which of the following treatment options is recommended for the patient shown in the image (Slide 1, Slide 2):
1) Halo-vest immobilization
3) Soft cervical collar
2) Minerva cast immobilization
5) Atlantoaxial arthrodesis
4) Open reduction and internal fixation
Treatment must address the fracture of the C 2 pars interarticularis and the C 2-C 3 facet subluxation. Options include traction to reduce the subluxation followed by external immobilization, or open reduction of the facet subluxation, which includes the opportunity to internally fix the fracture. Traction failed in this patient because the fracture eliminates a means to pull on the arch of C 2. If facet subluxation is not present, then a cervicothoracic orthosis such as a Philadelphia collar is adequate for healing of the C 2 spondylolysis.Correct Answer: Open reduction and internal fixation
2338. (3605) Q5-6487:
Flexion-type supracondylar fractures of the distal humerus in children are characterized by which of the following when compared to extension-type injuries:
1) Younger age at presentation
3) Higher risk of anterior interosseous nerve injury
2) Higher risk of ulnar nerve injury
5) Greater frequency of occurrence
4) Lower risk of needing open reduction
Flexion-type supracondylar fractures of the distal humerus in children include the following characteristics: Frequency is less than one-tenth as that of extension-type supracondylar fractures
Tend to occur in older children
Lower risk of anterior interosseous nerve injury Higher risk of ulnar nerve injury
More often require open reduction Correct Answer: Higher risk of ulnar nerve injury
2339. (3606) Q5-6488:
![]() |
![]() |
![]() |
slide 1 slide 2 slide 3
The 12-year-old boy shown in the radiographs (Slide 1, Slide 2) sustained his injury during a motor vehicle accident. Which of the following most accurately describes the injury:
1) Oblique proximal ulna metaphyseal fracture
3) Type 1 Monteggia fracture
2) Oblique proximal ulna fracture and elbow dislocation
5) Maisonneuve fracture
4) Type I Galeazzi fracture
This patient has a proximal ulna fracture and an anterior dislocation of the radial head. The ulna-trochlea relationship is intact. On the lateral film, the radial head does not articulate with the capitellum. On the anteroposterior film, the radial head points to the capitellum, but the joint space is absent in contrast to that of the ulna-trochlear joint, which is normal. This injury is a type 1 (anterior) Monteggia fracture. The dislocation is clearly visible on Slide 3.
A Galeazzi fracture refers to a distal radioulnar dislocation with a radial fracture. A Maisonneuve fracture is a related concept in the lower extremity: a proximal fibular fracture with an ankle mortise injury.
Correct Answer: Type 1 Monteggia fracture
2340. (3607) Q5-6489:
![]() |
![]() |
![]() |
![]() |
slide 1 slide 2 slide 3 slide 4
![]() |
slide 5
A 12-year-old boy sustained the injury shown (Slide 1, Slide 2, and Slide 3). A postreduction film is also presented (Slide 4). Which of the following treatments is the most likely to produce satisfactory results:
1) Accept reduction and continue immobilization for 6 weeks
3) Increase elbow extension and reapply cast
2) Increase pronation of forearm and reapply cast
5) Stabilize radiocapitellar joint with pin
4) Plate fixation of ulna
This patient has a type 1 (anterior) Monteggia fracture. The radial head is not reduced in the splint. Although supination and flexion may stabilize the radius, the radius remained unstable in this patient because it was dislocated with only a slight offset of the ulna. Plate fixation of the ulna is more likely to stabilize the radius in a patient with this injury. After the ulna is fixed with a plate (Slide 5), the radial head cannot be dislocated. Radiocapitellar pinning should be used only as a final option because of the associated risks of stiffness and breakage.Correct Answer: Plate fixation of ulna
2341. (3608) Q5-6490:
![]() |
![]() |
![]() |
slide 1 slide 2 slide 3
A 7-year-old boy is intubated after being involved in a motor vehicle accident. His lateral cervical film (Slide 1) reveals a:
1) Normal study
3) Hangmanâs fracture
2) Jefferson fracture
5) Subaxial subluxation
4) Occiput-C 2 subluxation
Extensive prevertebral soft tissue swelling is present. This patient received an injury that disrupted the ligaments of the atlanto-occipital and atlantoaxial joint. Additionally, he has a congenital synchondrosis of the C 1 ring. The dens-basion distance (dens to the anterior rim of the foramen magnum) is 13 mm; the normal distance is less than 10 mm to 12 mm. The patient is pentaplegic. Three-dimensional computed tomography scans (Slide 2, Slide 3) are presented.Correct Answer: Occiput-C 2 subluxation
2342. (3609) Q5-6491:
![]() |
![]() |
slide 1 slide 2
The images presented (Slide 1, Slide 2) depict which process:
1) Spondylolisthesis
3) Aneurysmal bone cyst
2) Spondylolysis (pars defect)
5) Degenerative disk disease
4) Herniated nucleus pulposus
The images presented depict a pars interarticularis fracture of the third lumbar vertebra.Correct Answer: Spondylolysis (pars defect)
2343. (3610) Q5-6492:
Patients who have had a unilateral slipped capital femoral epiphysis (SCFE) are at what odds of a contralateral slip compared to the general population:
1) 2 times
3) 200 times
2) 20 times
5) 20,000 times
4) 2,000 times
Patients who have presented with a unilateral SCFE are at approximately 2,000 times the risk of a contralateral slip compared to the general population. The risk of a contralateral slip in a juvenile is approximately 1/10,000, which provides justification for prophylactic pinning of a second side if a surgeon thinks it is warranted.Correct Answer: 2,000 times
2344. (3611) Q5-6493:
![]() |
![]() |
slide 1 slide 2
Which of the following conditions is represented in the clinical photographs (Slide 1, Slide 2):
1) Ectrodactyly
3) Macrodactyly
2) Constriction band syndrome
5) Clubfoot
4) Microdactyly
This patient has ectrodactyly, which is a condition characterized by the central absence of rays. Ectrodactyly, also known as cleft foot or lobster-claw, may affect the upper or lower extremities. The incidence is rare (approximately 1/90,000 live births). The condition may result from a defect in the apical ectodermal ridge. Associated anomalies are not always present but may include cleft hand, deafness, tibial hemimelia, or urinary tract anomalies. Patients with ectrodactyly have nearly normal function, as long as the foot is not so wide as to prohibit normal shoe wear. If shoe wear is difficult, then the foot can be reconstructed using flaps to close the cleft and metatarsal osteotomies as indicated.Correct Answer: Ectrodactyly
2345. (3612) Q5-6494:
![]() |
![]() |
slide 1 slide 2
A 6-year-old child is involved in a motor vehicle accident. What injury is demonstrated by the computed tomography (CT) (Slide 1) and magnetic resonance image (MRI) (Slide 2):
1) Jefferson fracture
3) Odontoid fracture
2) Transverse ligament injury
5) Pseudosubluxation
4) Hangmanâs fracture
This patient has an avulsion injury of the transverse ligament of the atlas. The bony avulsion is visible from its attachment to the left lateral mass and allows the atlas to move forward in relation to the axis. Hematoma and occiptocervical flexion is visible on the MRI. If it were an odontoid injury, the odontoid would still have a normal relationship to the ring of C 1 on the CT.
Treatment involves reduction and fusion of C 1-C 2. Correct Answer: Transverse ligament injury
2346. (3613) Q5-6495:
![]() |
![]() |
slide 1 slide 2
The radiograph (Slide 1) of an 11-year-old boy with significant leg pain is presented. No other abnormalities are found during the patientâs history or physical examination. The most likely diagnosis is:
1) Osteomyelitis
3) Enchondroma
2) Fibrous dysplasia
5) Ewing sarcoma
4) Unicameral bone cyst
Fibrous dysplasia is chronically expansile with vague transition to normal bone and a âground-glassâ appearance. These characteristics differentiate fibrous dysplasia from aneurysmal and unicameral bone cysts. Additionally, the lack of periosteal reaction differentiates fibrous dysplasia from malignancy or osteomyelitis. The patientâs magnetic resonance image (Slide 2) is also provided.Correct Answer: Fibrous dysplasia
2347. (3614) Q5-6496:
![]() |
![]() |
slide 1 slide 2
The parents of a 12-year-old boy notice an abnormal shape on their sonâs forearm. He states that he does not feel any pain. Based on his radiograph (Slide), the most likely diagnosis is:
1) Congenital dislocation of the radial head
3) Osteogenesis imperfecta
2) Unrecognized traumatic dislocation of the radial head
5) Multiple exostoses
4) Fibrous dysplasia
This patient has multiple exostoses. Multiple exostoses, which result from a mutation on one of three EXT genes, cause a growth disturbance of affected limb segments. This effect is most pronounced in the two-bone segments (forearm and lower leg). The bone with the exostoses grows less in length and often develops angulation. The less-affected bone often develops secondary deformity, such as the radial head dislocation in this patient.
Fibrous dysplasia and osteogenesis imperfecta may lead to developmental dislocation of the radial head as well, but the associated bony changes allow correct diagnosis of each of these conditions.
![]() |
![]() |
Correct Answer: Multiple exostoses 2348. (3615) Q5-6497:
slide 1 slide 2
This 12-year-old patient presents with limited forearm movement, but he does not have any associated pain. Based on his radiograph (Slide), recommended treatment includes:
1) Open relocation of radial head with annular ligament reconstruction (Bell-Tawse procedure)
3) Ulnar lengthening and closed or open radial head reduction
2) Radial head excision
5) Observation
4) Creation of single-bone forearm
This patient has multiple exostoses. The deformities apparent on his radiograph are typical for this condition: ulnar shortening and bowing, radial head dislocation, increased distal radial articular slope and negative ulnar variance, and altered distal radioulnar joint anatomy. These deformities are not present at birth but develop with growth. Although many of the deformities associated with multiple exostoses lead to loss of forearm rotation, the deformities are often not painful. Little evidence suggests that reconstruction of the morphologic abnormalities improves function or prevents pain. Therefore, most experts recommend that surgery be undertaken only when a symptom, such as painful prominence of an exostosis or pain at the radial head after skeletal maturity, can be relieved. Exostoses are more likely to be followed by recurrence or stiffness if âkissing lesionsâ are present on the radius and ulna.Correct Answer: Observation
2349. (3616) Q5-6498:
![]() |
![]() |
slide 1 slide 2
The 9-year-old patient presented in the radiographs (Slide 1, Slide 2) should be diagnosed with:
1) Congenital dislocation of the radial head
3) Osteogenesis imperfecta
2) Unrecognized traumatic dislocation of the radial head
5) Multiple exostoses
4) Fibrous dysplasia
This patient has type V osteogenesis imperfecta, which is characterized by hereditary radial head dislocation in 86% of patients (this patientâs mother and brother also have the condition bilaterally) as well as heterotopic calcification, osseous fragility, and scoliosis. Dislocation of the radial head occurs less often (0%-29%) in the other types of osteogenesis imperfecta and is linked to angular deformation of the long bones.
The signs of osteogenesis imperfecta in this patient include thin long bones, several healed fractures, and angular deformations. Correct Answer: Osteogenesis imperfecta
2350. (3617) Q5-6499:
Which has a lower risk of compartment syndrome or delayed detection in a patient with a supracondylar fracture of the humerus:
1) Median nerve injury
3) Ipsilateral forearm fracture
2) Flexion greater than 110°
5) Flexion-type mechanism of the fracture
4) Ecchymosis or severe swelling
Median nerve injury increases the risk of delayed detection of the fracture because of the lack of sensation in the volar compartment and also increases risk of injury to the adjacent brachial artery. Flexion greater than 110° increases forearm pressures by compromising arterial inflow and venous return. An ipsilateral forearm fracture is associated with increased risk of compartment syndrome (7% with ipsilateral forearm fracture vs 0.3%without ipsilateral forearm fracture). Severe ecchymosis and swelling are also associated with an increased risk of compartment syndrome, even in the presence of a radial pulse. A flexion-type supracondylar fracture, by contrast, has a lower risk of compartment syndrome because of the lack of stretch or tension on the critical vascular structures.Correct Answer: Flexion-type mechanism of the fracture
2351. (3618) Q5-6500:
In a lateral radiograph of a childâs elbow, the anterior humeral line should be located:
1) Anterior to the capitellum
3) In the middle-third of the capitellum
2) In the anterior one-third of the capitellum
5) Posterior to the capitellum
4) In the posterior-third of the capitellum
In a childâs elbow, the anterior humeral line should be located in the middle-third of the capitellum.Correct Answer: In the middle-third of the capitellum
2352. (3619) Q5-6501:
A 3-year-old boy falls on his upper extremity and shows sign of pain. His radiograph only shows a posterior fat pad sign of the elbow. What is the most likely diagnosis:
1) Nursemaid elbow
3) Supracondylar humerus fracture
2) Elbow sprain
5) Lateral condyle fracture
4) Proximal ulna fracture
A positive posterior fat pad sign is more suspicious for occult fracture than an anterior fat pad sign because flexing the elbow normally may elevate the anterior but not the posterior fat pad. A positive posterior fat pad sign is most likely to represent an undisplaced supracondylar fracture of the distal humerus, followed by proximal ulna, lateral conydle, and radial neck fracture. A nursemaid elbow is a diagnosis of exclusion and should be used only when the mechanism is one of traction rather than compression.Correct Answer: Supracondylar humerus fracture
2353. (3620) Q5-6502:
Which of the following is not an indication to urgently treat a supracondylar humerus fracture:
1) Antecubital ecchymosis
3) Associated forearm fracture
2) Firm compartments
5) Medial comminution
4) Poor perfusion
Medial comminution is an indication of potential bony instability but does not convey any increased vascular risk. All of the other indications convey a vascular risk, and treatment should not be delayed.Correct Answer: Medial comminution
2354. (3621) Q5-6503:
The lung attains half of its adult volume at what age:
1) 2 years
3) 6 years
2) 4 years
5) 10 years
4) 8 years
The lung attains 30% of its adult volume at 4 years and 50% at 8 years.Correct Answer: 8 years
2355. (3622) Q5-6504:
Which of the following conditions is not associated with an increased risk of cavus foot:
1) Freidreich ataxia
3) Charcot-Marie-Tooth disease
2) Spina bifida
5) Calcaneonavicular coalition
4) Tethered cord
Calcaneonavicular coalition is typically associated with a planovalgus foot. Freidreich ataxia, spina bifida, Charcot-Marie-Tooth disease, and tethered cord are often associated with a cavus foot.Correct Answer: Calcaneonavicular coalition
2356. (3623) Q5-6505:
Which of the following factors has not been proven to increase the risk of thermal necrosis to a limb with a cast:
1) Use of plaster
3) Folding a splint into double-thickness
2) Use of fiberglass
5) Using a dip water temperature more than 30°
4) Placing a recently casted limb on a pillow
Fiberglass is less likely than plaster to produce thermal injury, likely because of the increased porosity. All of the other factors listed increase the risk of thermal necrosis.Correct Answer: Use of fiberglass
2357. (3624) Q5-6506:
Which shape of the olecranon apophysis correlates most closely with closure of the triradiate cartilage of the hips:
1) Formation of initial ossification center
3) Formation of a curved single ossification center
2) Initial appearance of two ossification centers
5) Closure of the olecranon ossification center
4) Formation of a rectangular ossification center
The olecranon ossifies from two centers that merge and form one banana-shaped ossification center. When the ossificaiton center is rectangular in shape, it correlates with closure of the triradiate cartilage of the pelvis.Correct Answer: Formation of a rectangular ossification center
2358. (3625) Q5-6507:
A 15-year-old female patient undergoes spine fusion from T2 to the pelvis for scoliosis associated with cerebral palsy. Her past medical history is remarkable for seizure disorder, ventriculoperitoneal shunt, baclofen pump, and gastrostomy tube. Her postoperative course is complicated by disorientation and visual hallucinations on postoperative day 3, whereas she had been communicative preoperatively. The most likely cause of this change is:
1) Seizure activity
3) Sepsis
2) Postoperative anemia
5) Hydrocephalus
4) Baclofen withdrawal syndrome
This patient most likely has baclofen withdrawal due to interference with the catheter at some point along its path to the thecal sac. Baclofen withdrawal is characterized by increased spasticity, pruritus, hyperthermia, delusions, and hallucination. The pump reservoir should be checked, and a leak may be confirmed by filling the reservoir with a radionuclide and performing a scan.
Treatment involves repleting baclofen by oral or intrathecal means and fixing the leak.Correct Answer: Baclofen withdrawal syndrome
2359. (3626) Q5-6508:
A 5-year-old patient with Legg-Calvé-Perthes disease has lateral pillar B involvement. He has 15% epiphyseal extrusion. Treatment should consist of:
1) Observation
3) Iliac osteotomy
2) Femoral osteotomy
5) External fixation
4) Combined femoral and iliac osteotomy
Patients who have Legg-Calvé-Perthes disease with onset before 6 years of age have a good prognosis unless they have lateral pillar involvement in the B/C border or C categories.Correct Answer: Observation
2360. (3627) Q5-6509:
Which of the following characteristics is more commonly found in patients with Meyer dysplasia than in patients with Legg-Calvé-Perthes disease:
1) Bilateral involvement
3) Metaphyseal widening
2) Asynchronous onset
5) Later degenerative change
4) Epiphyseal extrusion
Meyer dysplasia is a rare condition that affects children before age 4. A characteristic of the condition includes delayed ossification of the epiphyses, which eventually develop normal ossification. Meyer dysplasia is synchronous, meaning that both heads have symmetrical involvement unlike Legg-Calvé-Perthes disease. Meyer dysplasia is usually bilateral, in contrast to Legg-Calvé-Perthes disease, which is bilateral in fewer than 10% of patients. There is no metaphyseal widening and no treatment is indicated.Correct Answer: Bilateral involvement
2361. (3733) Q5-7513:
A 10-year-old girl is seen for limb-length inequality. She has 10° of valgus of the left knee and a shortened left fibula. Her projected shortening at maturity is 1 cm in the left femur and 2.1 cm in the left tibia. Her left foot is normal in shape but one-half size smaller than the right. Her overall height and maturity are at the median for her age. For this patient, recommended treatment for limb-length inequality is:
1) Observation
3) Epiphysiodesis of the right tibia
2) Epiphysiodesis of the left tibia
5) Lengthening of the left tibia
4) Shortening of the right tibia
This patient has a projected discrepancy at maturity of 3.1 cm. Most experts agree that the risk of later gait disturbance or back pain is increased above a normal threshold when limb length inequality exceeds 2.5 cm. A shoe lift is noninvasive but not likely to be permanently adopted. Epiphysiodesis of the right tibia is the least invasive and least complicated option for permanent equalization and should be performed in this patient. Shortening of the right tibia would be an option if the patient desired correction after skeletal maturity, but is a more invasive procedure with risk of compartment syndrome, nonunion, and pain.
Lengthening of the left tibia is a valid option, which could also allow correction of the valgus and maintenance of stature. However, the procedure leaves scars and complications are more significant. Most surgeons would not think that it is worthwhile for gain of an inch because a left tibial hemiepiphysiodesis could easily correct the valgus.
Correct Answer: Epiphysiodesis of the right tibia
2362. (3739) Q5-7519:
Which of the following is an accurate mean age for a motor milestone:
1) Sit up without assistance at 6 months
3) Hop on one foot at 2 years
2) Walk independently at 24 months
5) Able to run at 36 months
4) Roll from prone to supine position at 2 months
Motor milestones provide a way of assessing a childâs motor development. If they are not met, a physician or parent may be more alert for neurological or connective tissue disorders.
Common pediatric milestone include:
Sit up without assistance at 6 months Walk independently at 12 months Hop on one foot at 4 years
Roll from prone to supine position at 4 months Able to run at 18 months
Correct Answer: Sit up without assistance at 6 months
2363. (3740) Q5-7520:
In the workup of scoliosis, what is/are the best method(s) to assess skeletal maturity during a patientâs peak height velocity (Risser grade 0):
1) Greulich and Pyle atlas (hand/wrist radiographs)
3) Tanner-Whitehouse-III RUS (radius, ulna, small bones of hand) method
2) Modified Sauvegrain method (olecranon radiographs)
5) B, C, and D
4) Triradiate cartilage closure
Precise information about skeletal maturity during peak growth rate is important in deciding treatment options in patients with scoliosis. The modified Sauvegrain method, Tanner-Whitehouse-III RUS method, and triradiate cartilage closure are each accurate and reliable markers of maturity that the orthopedic surgeon can use to quickly assess pediatric patients.
Correct Answer: B, C, and D
2364. (3741) Q5-7521:
Following triple arthrodesis, total ankle range of motion is:
1) Not affected because the tibiotalar joint is not fused in a triple arthrodesis
3) Is increased in plantarflexion only; dorsiflexion is unchanged
2) Is increased because it must compensate for the loss of motion through the hindfoot joints
5) Is decreased in plantarflexion only; dorsiflexion is unchanged
4) Is decreased in both dorsiflexion and plantarflexion
Total ankle motion is a combination of tibiotalar motion as well as motion in the hindfoot joints. A 10-year follow-up study by Smith and colleagues evaluated the change in range of motion following triple arthrodesis. The authors found a 27% loss of ankle plantarflexion following triple arthrodesis, but no loss of dorsiflexion. Despite this loss of motion, 93% of patients were satisfied at average 10-year follow-up.
Correct Answer: Is decreased in plantarflexion only; dorsiflexion is unchanged
2365. (3824) Q5-7607:
Three years ago, a 20-year-old female patient underwent posterior spinal fusion for adolescent idiopathic scoliosis. She asks what would happen if the implants were removed. The surgeonâs most likely response will be:
1) The spine will not change alignment, if the fusion is solid.
3) The scoliosis is likely to increase, even if the fusion is solid.
2) The sagittal alignment will likely change, even if the fusion is solid.
5) Increased back pain is likely.
4) Fracture of the fusion mass is likely.
After removal of implants, the sagittal alignment is likely to change, with kyphosis increasing by 10º in nearly half of all patients and by more than 20º in 10% of patients. By contrast, scoliosis increases significantly in only 5% of patients. Most patients who have implants removed experience an improvement in symptoms. Fracture of the fusion mass is rare.
Correct Answer: The sagittal alignment will likely change, even if the fusion is solid.
2366. (3825) Q5-7608:
Which of the following factors best predicts the risk of disk degeneration in young adults:
1) Smoking history
3) Gender
2) Occupational exposure
5) Participation in sports
4) Family history
Genetic factors are most predictive of disk degeneration. However, various genes have been identified as possible causative factors, possibly operating in different ways and interacting with environmental and other factors.
Correct Answer: Family history
2367. (3826) Q5-7609:
Which of the following treatments decreases hip subluxation in patients with cerebral palsy who have displaced hips:
1) Passive stretching
3) Hip abduction bracing and botulinum toxin type A
2) Hip abduction bracing
5) Surgical adductor lengthening
4) Electrical stimulation of the hip abductors
Only hip adductor lengthening has been shown to decrease hip subluxation in patients with displaced hips. Patients treated with botulinum toxin type A and hip abduction still result in progression.
Correct Answer: Surgical adductor lengthening
2368. (3827) Q5-7610:
Which of the following groups is not at increased risk for community-acquired methicillin-resistant Staphylococcus aureus (CA-MRSA):
1) Athletes who participate in physical contact sports
3) People traveling abroad
2) Patients who have had recent antibiotics
5) Prison inmates
4) People living in crowded conditions
People traveling abroad are not considered at increased risk for CA-MRSA. Athletes who participate in physical contact sports, patients who have had recent antibiotics, people living in crowded conditions, and prison inmates are considered at increased risk for CA-MRSA.
Correct Answer: People traveling abroad
2369. (3828) Q5-7611:
Which of the following toxins is responsible for the virulence of community-acquired methicillin-resistant Staphylococcus aureus (CA-MRSA):
1) Beta-lactam
3) Oxazolidine
2) Panton-Valentine leukocidin
5) Streptolysin
4) mecA
Panton-Valentine leukocidin is a toxin that produces necrosis of tissue and white blood cells. This toxin is much more common in CA-MRSA than in hospital-acquired MRSA.
Correct Answer: Panton-Valentine leukocidin
2370. (3829) Q5-7612:
A 5-year-old girl presents to the emergency department with pain in her hip. Her temperature is 102° F, pulse is 96, and blood pressure is 104/60. Magnetic resonance imaging reveals edema in the obturator muscles. Blood cultures are negative.
Ultrasound-guided aspiration reveals no abscess, but fluid obtained grows methicillin-resistant Staphylococcus aureus (MRSA). In addition to antibiotic treatment, what other treatment option is recommended:
1) Surgical irrigation and debridement
3) Repeated aspiration with ultrasound guidance
2) Indwelling catheter drainage
5) Spica cast application
4) Observation
This patient has community-acquired MRSA but does not have an evident fluid collection. There is no evidence of systemic infection or necrotizing fasciitis. There is no indication for drainage or debridement of any type. Therefore, antibiotics alone with clinical observation are indicated.
Correct Answer: Observation
2371. (3830) Q5-7613:
A 5-year-old girl who presents to the emergency department has obturator muscle infection without abscess formation, which is seen on magnetic resonance imaging. She has no clinical evidence of sepsis. Aspiration yields methicillin-resistant Staphylococcus aureus (MRSA). Which of the following antibiotics is recommended:
1) Vancomycin
3) Rifampin
2) Clindamycin
5) Linezolid
4) Tetracycline
To treat this patient, clindamycin is the drug of choice. Vancomycin is not recommended for uncomplicated MRSA in the nonseptic patient. Rifampin is not recommended to be used alone because of the rapid development of resistance. Tetracycline is not recommended for children under 8 years old because it causes permanent staining of the teeth. Linezolid is to be used only after infectious disease consultation because of its toxicity (thrombocytopenia in 3.5% of patients) and expense. Another option for this patient would be trimetoprim/sulfamethoxazole.
Correct Answer: Clindamycin
2372. (3831) Q5-7614:
A nonverbal 12-year-old patient with totally involved cerebral palsy and developmental delay has been in pain for the past week. The source of the pain cannot be localized. Pelvis radiographs and urinalysis are unremarkable. The next recommended study is:
1) Ultrasound of the hips
3) Whole-body bone scan
2) Magnetic resonance imaging (MRI) of the brain
5) Thermography
4) Computed tomography (CT) of the lumbar spine
In a nonverbal child with longstanding pain, the whole-body bone scan is the most sensitive and specific test. Whole-body bone scans detected a cause of pain in more than half of such children in one study; most causes of pain were fractures. MRI cannot be focused enough, and CT has the same limitation as well as increased radiation exposure. Thermography is not widely available and has not been studied in children with cerebral palsy.
Correct Answer: Whole-body bone scan
2373. (3832) Q5-7615:
A 2-month-old infant has a midshaft humeral fracture. The fracture has 45° angulation with an anterolateral apex and is 6 mm short. Recommended treatment includes:
1) Immobilizing the arm against the torso
3) Percutaneous pin fixation
2) A shoulder abduction cast
5) External fixation
4) Plate fixation
The humerus has unlimited remodeling potential at this age. The goal of treatment is pain relief. Immobilization against the torso is the most practical method of treatment.
Correct Answer: Immobilizing the arm against the torso
2374. (3833) Q5-7616:
A 23-month-old patient demonstrates intoeing. He walked at 21 months. His height is in the 3rd quantile, and he is cognitively normal. Possible diagnoses include all of the following except:
1) Osteogenesis imperfecta
3) Internal tibial torsion
2) Rickets
5) Metaphyseal chondrodysplasia
4) Achondroplasia
The upper limit of normal age for onset of walking is 18 months. All of the diagnoses, except for internal tibial torsion, may explain the delayed walking, short stature, and intoeing.
Correct Answer: Internal tibial torsion
2375. (3834) Q5-7617:
The most common form of rickets in the United States is:
1) Nutritional deficiency of calcium
3) Vitamin D-dependent rickets (1-alpha-hydroxylase deficiency)
2) Hypophosphatasia
5) X-linked hypophosphatemic rickets
4) Fanconi syndrome
Although rickets due to deficiency of vitamin D is common due to inadequate sunlight or prolonged breast feeding, nutritional deficiency of calcium is uncommon in the US. The most common cause of rickets in the US is X-linked hypophosphatemic rickets. Hypophosphatasia, caused by alkaline phosphatase deficiency, is a rare recessive condition. Fanconi syndrome is a failure of tubular reabsorption of small molecules and resembles X-linked hypophosphatemic rickets, although it is more rare. Vitamin D-dependent rickets is also rare.
Correct Answer: X-linked hypophosphatemic rickets
2376. (3835) Q5-7618:
The gene most likely to be associated with inherited rickets is:
1) PHEX
3) SLC 34A3
2) FGF23
5) COL1A1
4) EXT1
PHEX is a gene whose product regulates transport of renal phosphates that leads to isolated renal phosphate wasting and consequent hypophosphatemia. This is the syndrome known as X-linked hypophosphatemic rickets, which is the most common cause of inherited rickets.
Correct Answer: PHEX
2377. (3836) Q5-7619:
Genetic testing performed for a patient reveals a defect in PHEX. The family history provided by the patient is most likely to include:
1) An affected father and son in the family
3) A negative family history for the disorder
2) More females relatives affected than male relatives
5) Affected siblings but no prior ancestors or relatives
4) An equal number of affected male relatives and females relatives
PHEX is the gene for a product that regulates renal tubular phosphate reabsorption. A defect in this gene causes X-linked hypophosphatemic rickets, which is the most common inherited rickets. It is an X-linked dominant condition; therefore, male-to-male transmission cannot occur. An excess of affected female relatives to male relatives is approximately 2:1. Spontaneous mutation can occur but is less common than inheritance of the disorder.
Correct Answer: More females relatives affected than male relatives
2378. (3837) Q5-7620:
A toddler with bowing is suspected of X-linked hypophosphatemic rickets (XLH). The most likely laboratory abnormalities will include:
1) Low alkaline phosphatase
3) Low calcium
2) High PO4
5) Low 1,25-(OH)2D3
4) Low 25-(OH)2D3
The most common laboratory abnormalities in XLH include high alkaline phosphatase (in attempt to mobilize phosphate from the bone); low serum phosphate; normal calcium and 25-(OH)2D3; and low 1,25-(OH)2D3, presumably due to defective renal hydroxylation.
Correct Answer: Low 1,25-(OH)2D3